87
KAPLAN LSAT PREP LSAT RELEASED TEST I EXPLAINED A Guide to the June, 1991 LSAT KAPLAN The answer to the test question.

LSAT - dl.keywin.orgdl.keywin.org/a/a/aafd9027c6dbefbc457fbda910414daf.pdf · KAPLAN LSAT PREP LSAT RELEASED TEST I EXPLAINED A Guide to the June, 1991 LSAT KAPLAN The answer to the

Embed Size (px)

Citation preview

Page 1: LSAT - dl.keywin.orgdl.keywin.org/a/a/aafd9027c6dbefbc457fbda910414daf.pdf · KAPLAN LSAT PREP LSAT RELEASED TEST I EXPLAINED A Guide to the June, 1991 LSAT KAPLAN The answer to the

KAPLAN LSAT PREP

LSAT

RELEASED TEST IEXPLAINED

A Guide to the June, 1991 LSAT

KAPLANThe answer to the test question.

Page 2: LSAT - dl.keywin.orgdl.keywin.org/a/a/aafd9027c6dbefbc457fbda910414daf.pdf · KAPLAN LSAT PREP LSAT RELEASED TEST I EXPLAINED A Guide to the June, 1991 LSAT KAPLAN The answer to the

„1996 Stanley H. Kaplan Educational Center Ltd

All rights reserved. No part of this book may be reproduced in any form, byphotostat, microfilm, xerography or any other means, or incorporated into anyinformation retrieval system, electronic or mechanical, without the written permissionof Stanley H. Kaplan Educational Center Ltd.

Page 3: LSAT - dl.keywin.orgdl.keywin.org/a/a/aafd9027c6dbefbc457fbda910414daf.pdf · KAPLAN LSAT PREP LSAT RELEASED TEST I EXPLAINED A Guide to the June, 1991 LSAT KAPLAN The answer to the

© K A P L A N 1

SECTION I:

READING COMPREHENSION

Page 4: LSAT - dl.keywin.orgdl.keywin.org/a/a/aafd9027c6dbefbc457fbda910414daf.pdf · KAPLAN LSAT PREP LSAT RELEASED TEST I EXPLAINED A Guide to the June, 1991 LSAT KAPLAN The answer to the

LSAT PREP __________________________________________________________________ LSAT Test I Explained: Section I

2 © K A P L A N

Passage 1—Phillis Wheatley(Q. 1-8)

Topic and Scope: Phillis Wheatley, specifically Wheatley’s poetry.

Purpose and Main Idea: The author’s purpose is to discuss and evaluate Wheatley’spoetry. The author’s specific main idea is that Wheatley was an accomplished poet whorightly deserves to be known as the first African American poet, though her poetry was notinfluenced by African traditions and did not contribute to the growth of an AfricanAmerican literary tradition.

Paragraph Structure: ¶1 states that Wheatley quickly became an accomplished poet, eventhough she was not raised in an English-language environment. ¶2 digresses from the topicin order to discuss the background of African American English.

¶3 reveals the author’s belief that Wheatley’s poetry was not influenced by her Africanheritage. ¶4 summarizes the author’s feelings about Wheatley: though she did notcontribute to the growth of an African American literary tradition, she was nevertheless anaccomplished poet who deserves to be recognized as the first African American poet.

The Big Picture:

• Topic and scope are evident early on in the passage, but purpose and main idea arenot. Instead, the passage hits you with a lot of details. It’s not until ¶s 3 and 4 thatyou get a strong sense of authorial purpose and main idea. On Test Day, it might bebetter to leave a similarly structured passage for later in the section. In general, it’sbest to begin your work on the Reading Comprehension section with a passagewhose purpose and main idea (if there is one) are apparent early in the text.

• Don’t assume that all the key information is contained in the first ¶. In this passage,for instance, most of the important information emerges toward the end. Alwaysapply your critical reading skills to the entire passage.

The Questions:

1. (C)This choice nicely captures the author’s main idea. Although Wheatley was anaccomplished poet, her poetry did not reflect the traditions of her African heritage.

(A) is beyond the scope of the text. This passage is specifically about Wheatley’s poetry. Itdoesn’t directly compare her poetry to the work of folk artists.

(B) is a “half right, half wrong” choice. The first part of this choice is okay, but the secondgoes awry in suggesting that Wheatley was influenced by African traditions.

(D) and (E) According to the author, Wheatley’s poetry was disconnected from Africantraditions and African American literary developments.

Page 5: LSAT - dl.keywin.orgdl.keywin.org/a/a/aafd9027c6dbefbc457fbda910414daf.pdf · KAPLAN LSAT PREP LSAT RELEASED TEST I EXPLAINED A Guide to the June, 1991 LSAT KAPLAN The answer to the

LSAT PREP __________________________________________________________________ LSAT Test I Explained: Section I

© K A P L A N 3

• The correct answer to a “global” question must be broad enough to encompass thecontents of the entire passage, yet narrow enough to refer to the specific subjectmatter of the passage.

• Often, the wrong answers to global questions are in direct conflict with the author’spoint of view. So when you see a choice that represents the opposite of your pre-phrase, don’t panic. You’re probably on the right track. Expect to see a few aucontraire answer choices, and you’ll eliminate them with confidence.

2. (E)According to the passage, Wheatley’s poetry was influenced by the English literaryconventions of her day. Her poetry was neither influenced by nor reflected her ethnicheritage. A contemporary Italian immigrant poet who is influenced solely by theconventions of modern American poetry would be following a similar approach to poetryas Wheatley.

(A), (B), (C), (D) All of these choices go wrong in suggesting that Wheatley’s work wassomehow influenced by or reflected her ethnic heritage.

• In questions that ask you to compare hypothetical situations to a situation describedin the passage, look for the choice that mimics the relationship outlined in thepassage.

3. (D)Lines 19-24 make precisely this point.

(A) and (B) The passage mentions African American religious music and folk art, but notthe religious music and folk art of New England colonists. New England coloniststhemselves, as a matter of fact, aren’t even mentioned in the passage.

(C) According to lines 19-24, African languages affected spoken English, not formal writtenEnglish.

(E) The passage doesn’t discuss general “eighteenth-century aesthetic principles,” andthere’s nothing in the passage to suggest that African languages had an impact on them.

• If you’re in doubt about a detail, be sure to check back with the passage, using your“roadmap” of the passage to locate the relevant text. Don’t answer on a hunch or avague recollection.

Page 6: LSAT - dl.keywin.orgdl.keywin.org/a/a/aafd9027c6dbefbc457fbda910414daf.pdf · KAPLAN LSAT PREP LSAT RELEASED TEST I EXPLAINED A Guide to the June, 1991 LSAT KAPLAN The answer to the

LSAT PREP __________________________________________________________________ LSAT Test I Explained: Section I

4 © K A P L A N

4. (D)The phrase “closed system” emerges in the context of the author’s remark that eighteenth-century English poetry did not incorporate the informal, popular language of its day.Hence, by a closed system of poetry the author undoubtedly means poetry that “does notadmit the use of street language and casual talk.”

(A) If anything, the author believes that poetry can be written by someone who isn’t raisedknowing its conventions. After all, as the author points out, Wheatley wrote poetry basedon conventions that she learned quickly, after she had reached New England.

(B) and (E) are beyond the scope of the text. The author never discusses poetry’s impact onspoken language (B). Nor does she discuss why poetry might ultimately be rejected (E).

(C) In lines 35-36, the author says that poetry in a “closed system” is based on imitatingpast works.

• Whenever a question asks you to interpret the meaning of a word or phrase, read thelines around that word or phrase. The key to understanding what the author meansby a word or phrase lies in understanding the context in which it appears.

5. (A)Line 41 states that the conventions of eighteenth-century English poetry permitted the poetto express “generalized feelings.”

(B), (D), (E) The conventions of this poetry, according to line 40, did not allow “casualtalk” (D). The ban on casual talk, of course, would also have eliminated “AmericanizedEnglish” (B) and “Black speech” (E). Indeed, their absence from Wheatley’s poetry isconfirmed by lines 36-38.

(C) The lack of “themes from folk art” in Wheatley’s poetry is highlighted in lines 42-49.

• This question illustrates the importance of having an awareness of paragraphstructure. If you kept in mind where in the passage the author evaluates Wheatley’spoetry, you should have gone directly to ¶3 for the answer to this question.

6. (E)In lines 54-56, the author claims that “Wheatley’s poetry contributed little to thedevelopment of a distinctive African American literary language.” Thus, if scholars wereto “trace themes and expressions in African American poetry back to the poetry ofWheatley,” the author’s argument concerning Wheatley’s role in the evolution of anAfrican American literary language would be seriously weakened.

(A) Since the author claims that Wheatley’s poetry conformed to the conventions ofneoclassical poetry, her argument would not be undermined by a finding that Wheatley’spoetry was admired in England because of its adherence to English literary traditions.

Page 7: LSAT - dl.keywin.orgdl.keywin.org/a/a/aafd9027c6dbefbc457fbda910414daf.pdf · KAPLAN LSAT PREP LSAT RELEASED TEST I EXPLAINED A Guide to the June, 1991 LSAT KAPLAN The answer to the

LSAT PREP __________________________________________________________________ LSAT Test I Explained: Section I

© K A P L A N 5

(B) Why would the author’s argument about Wheatley’s poetry be weakened by thediscovery that Wheatley had written a history of her family’s experiences?

(C) Likewise, why would the author’s argument about Wheatley’s poetry be weakened bythe knowledge that other Africans were able to overcome the language barriers that sheovercame?

(D) misrepresents the author’s criticism. The author argues that Wheatley failed to play ameaningful role in the development of an African American literary language. WhetherWheatley was an important poet in the American literary tradition is another issuealtogether.

• In questions that ask you to pick the choice that would most strengthen or weakenan authorial position, read the question stem carefully to be sure which you’re askedto do. Often, an incorrect choice will do the exact opposite of what you’re asked todo, and you could fall for this trap choice if you haven’t read the question stemcarefully.

• Don’t bend over backwards for an answer choice. If the relevance of a choicedepends upon making assumptions unsupported by the passage, then that choice isirrelevant. For example, you could spend precious time on choice (B) attempting tofind a connection between a family history and the evolution of an AfricanAmerican literary language, but what would be the point?

7. (D)In lines 30-31, the author expresses dismay that Wheatley did not integrate Africantraditions into her poetry in order to create an African American literary language. It isinferable, therefore, that the author would have praised Wheatley’s work even more hadWheatley done so.

(A) “Black English” is mentioned in the third ¶ as a potential source that Wheatley couldhave used in creating an African American literary language. However, the author nevercriticizes Wheatley for failing to influence the way English is spoken.

(B) The author would have praised Phillis Wheatley more if she had invented an AfricanAmerican literary tradition.

(C) To the extent that the author is critical of Wheatley, it’s because Wheatley so strictlyadhered to the conventions of eighteenth-century English poetry.

(E) The author doesn’t bring up the general issue of “the barriers that written Englishliterary forms presented to Black authors.”

• This question illustrates the importance of grasping authorial main idea. If yourecognized that the author argues that Wheatley failed to play a role in developing anAfrican American literary language, settling on choice (D) should have been a snap.

Page 8: LSAT - dl.keywin.orgdl.keywin.org/a/a/aafd9027c6dbefbc457fbda910414daf.pdf · KAPLAN LSAT PREP LSAT RELEASED TEST I EXPLAINED A Guide to the June, 1991 LSAT KAPLAN The answer to the

LSAT PREP __________________________________________________________________ LSAT Test I Explained: Section I

6 © K A P L A N

8. (B)While the author praises Wheatley as an accomplished poet, she also notes that Wheatleyfailed to contribute to the African American literary tradition. Thus, the author’s attitudetoward Wheatley’s work is aptly characterized as one of “qualified admiration.”

(A) “Enthusiastic advocacy” suggests that the author is uncritical of Wheatley’s work.

(C) and (D) “Dispassionate impartiality” (C) and “detached ambivalence” (D) suggest thatthe author has no real opinion about Wheatley’s work.

(E) “Perfunctory dismissal” suggests that the author condemns Wheatley’s work.

• Watch out for extreme-sounding choices such as (A) and (E). Most LSAT ReadingComprehension passages are more moderate in tone.

Page 9: LSAT - dl.keywin.orgdl.keywin.org/a/a/aafd9027c6dbefbc457fbda910414daf.pdf · KAPLAN LSAT PREP LSAT RELEASED TEST I EXPLAINED A Guide to the June, 1991 LSAT KAPLAN The answer to the

LSAT PREP __________________________________________________________________ LSAT Test I Explained: Section I

© K A P L A N 7

Passage 2—Cell Biology(Q. 9-16)

Topic and Scope: The relationship between cell biology (cytology) and biochemistry;specifically, how the two disciplines have affected each other over the last century.

Purpose and Main Idea: The author’s purpose is to describe and evaluate the historicalinteraction between cell biology and biochemistry. The author’s main idea is to argue thatthe interaction between these two disciplines has lead to progress in both of them.

Paragraph Structure: ¶1 notes that two scientific disciplines concerned with the samegeneral topic are often in conflict with each other in their early stages of developmentbecause of their different approaches to the topic. One discipline takes a “macro”approach to the topic, while the other discipline takes a “micro” approach.

¶s 2 and 3 go on to describe a particular historical case of conflict between two relatedscientific disciplines: cell biology and biochemistry. ¶4 explains that the claims of bothdisciplines have essentially been borne out, leading to a synthesis of the two in the form ofthe new discipline of molecular genetics.

¶5 summarizes the author’s main idea: competition between these related scientificdisciplines has been useful, for it has generated important scientific discoveries.

The Big Picture:

• Whenever a passage compares/contrasts entities—as this passage does—make surethat you can distinguish between (among) the entities. The questions will certainlytest if you can tell them apart.

• Don’t let unfamiliar scientific terms or concepts throw you. As you read, boil ascience passage’s ideas down to their essence. If you simplify things as you go, thequestions will be much less intimidating.

The Questions:

9. (B)After reviewing (in ¶s 1-3) the process by which related scientific disciplines interact, ¶s 4and 5 go on to argue that this interaction is often scientifically beneficial.

(A) Neither the author’s general discussion of the interaction between related scientificdisciplines nor his specific example of the interaction between cell biology andbiochemistry suggests that this interaction has negative consequences for the advancementof science.

(C) The author doesn’t go so far as to contend that the interaction between relateddisciplines necessarily leads to a synthesis of their approaches in all cases. After all, he onlylooks at one case.

Page 10: LSAT - dl.keywin.orgdl.keywin.org/a/a/aafd9027c6dbefbc457fbda910414daf.pdf · KAPLAN LSAT PREP LSAT RELEASED TEST I EXPLAINED A Guide to the June, 1991 LSAT KAPLAN The answer to the

LSAT PREP __________________________________________________________________ LSAT Test I Explained: Section I

8 © K A P L A N

(D) This choice focuses on details in ¶s 2 and 3.

(E) This choice focuses on a detail in ¶5.

• In “global” questions, watch out for choices that focus on one part of the passagerather than on the entire passage.

10. (E)This choice nicely paraphrases lines 19-21.

(A) Line 39 mentions “chromosome mapping,” but suggests that this may not have been alate nineteenth-century development. Moreover, it’s not clear whether cytologists orbiochemists did the mapping.

(B) and (D) According to lines 29-32, late nineteenth-century biochemists, not cytologists,were concerned with the “chemical nature of protoplasm” (B) and the “role of enzymes inbiological processes” (D).

(C) According to ¶4, biochemists, not cytologists, deal with the “spatial relationship ofmolecules within the cell.” Moreover, this issue has been the focus of post-1950biochemists.

• This question illustrates the importance of checking back with the passage aboutdetails. All of the choices might have looked very tempting if you didn’t consult ¶s2-4.

11. (A)If biochemists “stood apart” from the late nineteenth-century debate about the structuralnature of protoplasm (as lines 25-29 say), then it stands to reason that the debate must havetaken place among cytologists.

(B), (C), (E) Again, lines 25-29 say that late nineteenth-century biochemists avoided “thedebate over the structural nature of protoplasm.”

(D) is out for the simple reason that the passage never even mentions “geneticists,” let alonelate nineteenth-century geneticists.

• Sometimes questions can be answered through a process of elimination. If it wasn’timmediately clear to you that the debate took place among cytologists, you couldhave eliminated choices (B)-(E) by recognizing that neither biochemists norgeneticists were involved in the debate.

Page 11: LSAT - dl.keywin.orgdl.keywin.org/a/a/aafd9027c6dbefbc457fbda910414daf.pdf · KAPLAN LSAT PREP LSAT RELEASED TEST I EXPLAINED A Guide to the June, 1991 LSAT KAPLAN The answer to the

LSAT PREP __________________________________________________________________ LSAT Test I Explained: Section I

© K A P L A N 9

12. (B)This choice nicely paraphrases lines 35-37.

(A) The chemical reactions that occur in cytological preparations (lines 23-24) were anissue raised by late nineteenth-century biochemists, not late nineteenth-century cytologists.

(C) According to lines 29-31, late nineteenth-century biochemists were interested in the“nature of protoplasm.”

(D) The only mention of “cell division” occurs in lines 20-21, and this reference hasnothing to do with the criticism of late nineteenth-century biochemists leveled by latenineteenth-century cytologists.

(E) Late nineteenth-century cytologists criticized the methods, not the knowledge, of latenineteenth-century biochemists.

• A common “trap” choice is the answer choice that uses the passage’s language butdistorts the passage’s ideas. Such choices are especially common in sciencepassages, which often contain complex language and unfamiliar terminology.

13. (A)The first sentence of ¶5 contains the author’s main idea: “[The] interaction between paireddisciplines can have important [scientific] results.” A few lines later, the author quotes asimilar remark by Fruton. The author, in other words, quotes Fruton in order to reinforcethe author’s own conclusion.

(B), (C), (E) Fruton’s remark is general in nature; it doesn’t refer specifically to cytology orbiochemistry.

(D) Like the author, Fruton believes that competition between disciplines generatesprogress, not problems.

• Wrong choices are often wrong for the same reason. In this case, note that choices(B), (C), and (E) all incorrectly claim that Fruton’s remark applies to the specific caseof cytology and biochemistry.

14. (A)¶2 indicates that the enzyme theory emerged in the late nineteenth century, while ¶4indicates that the discipline of molecular biology is mainly a post-1950 phenomenon. Inother words, the enzyme theory appeared before molecular biology.

(B) According to ¶2, the initial discovery of cell architecture, like the formulation of theenzyme theory, occurred in the late nineteenth century.

(C) ¶3 suggests that chromosome mapping may have occurred later than the latenineteenth century. In any case, it certainly didn’t occur before the late nineteenth century.

Page 12: LSAT - dl.keywin.orgdl.keywin.org/a/a/aafd9027c6dbefbc457fbda910414daf.pdf · KAPLAN LSAT PREP LSAT RELEASED TEST I EXPLAINED A Guide to the June, 1991 LSAT KAPLAN The answer to the

LSAT PREP __________________________________________________________________ LSAT Test I Explained: Section I

10 © K A P L A N

(D) The synthesis of ideas between cytologists and biochemists is another post-1950phenomenon.

(E) Lines 25-32 suggest that the debate about the nature of protoplasm emerged before theenzyme theory.

• The correct answers to “inference” questions stick closely to the spirit of the text. Ifyou have to go through an intricate reasoning process in order to justify a choice,you’ve probably picked an incorrect choice.

15. (B)Lines 29-31 state that late nineteenth-century biochemists were concerned with thechemical nature of protoplasm. Hence, it’s inferable that they considered understandingthe nature of protoplasm to be vital to understanding cell processes.

(A) and (D) Late nineteenth-century cytologists, not late nineteenth-century biochemists,were concerned with the structure of the cell (A) and the physical traits of protoplasm (D).

(C) and (E) are beyond the scope of the passage. There’s no mention of what late nineteenth-century biochemists may have thought about the “behavioral patterns of organisms” (C).Nor is there any discussion about whether they thought chemistry was inadequate for thetask of understanding the cell (E).

• It’s important to read the question stem carefully. This particular question asksabout the views of biochemists, not cytologists. If you missed this point because youread the stem too quickly, you might have fallen for wrong choice (A) or (D).

16. (C)¶1 presents a general proposition regarding the relationship between two similar scientificdisciplines. The rest of the passage illustrates this proposition through an extendedexample: the relationship between cytology and biochemistry as it has developed over thelast century.

(A) What process? What reason for its occurrence?

(B) What set of examples? Only one example—the relationship between cytology andbiochemistry—is explored.

(D) What statement of principles? What rationale?

(E) What problem? What solution?

• Questions with abstractly-worded answer choices aren’t necessarily more difficultthan questions with concretely-worded answer choices. Don’t sacrifice an easy pointout of an irrational fear of abstract language.

Page 13: LSAT - dl.keywin.orgdl.keywin.org/a/a/aafd9027c6dbefbc457fbda910414daf.pdf · KAPLAN LSAT PREP LSAT RELEASED TEST I EXPLAINED A Guide to the June, 1991 LSAT KAPLAN The answer to the

LSAT PREP __________________________________________________________________ LSAT Test I Explained: Section I

© K A P L A N 11

Passage 3—Criminal Procedure(Q. 17-21)

Topic and Scope: The adversarial and inquisitorial systems of criminal procedure;specifically, the differences between these two forms of criminal procedure.

Purpose and Main Idea: The author’s purpose is to compare/contrast the two systems ofcriminal procedure. Although the author largely restricts himself to a factual descriptionof the two systems, he also argues that the inquisitorial system is superior to the adversarialsystem.

Paragraph Structure: ¶1 introduces the two systems of criminal procedure: adversarialand inquisitorial. ¶s 2 and 3 describe the adversarial and inquisitorial systems,respectively. ¶s 4 and 5 point out some of the differences between the two systems.

The Big Picture:

• Whenever the passage compares multiple theories or approaches, make sure thatyou’re clear about their differences. The questions will test to see whether you’vepicked up on them.

• In passages that contain a lot of intricate details—passages like this one—it’s amistake to try to assimilate the details during a first read through of the text. Instead,note where in the passage each detail occurs. Doing so will allow you to quickly lookup any detail cited by the questions.

• Don’t give up on harder passages on the assumption that no easy points areassociated with them. Often, a difficult passage will have very accessible questions(as is the case here).

The Questions:

17. (A)Lines 47-48 point out that the adversarial system is based on legal rules.

(B) To the extent that reenactments of crimes are associated with either system of criminalprocedure, they are more closely identified with the inquisitorial system, as lines 48-50suggest.

(C) Lines 44-45 point out that the inquisitorial system is based on a search for the facts.

(D) Personal vengeance is a part of neither the adversarial nor the inquisitorial systems.These systems, as a matter of fact, replaced the earlier system of private vengeance.

(E) Police testimony has a role to play in both the adversarial and the inquisitorial systems.It’s not what distinguishes one from the other.

Page 14: LSAT - dl.keywin.orgdl.keywin.org/a/a/aafd9027c6dbefbc457fbda910414daf.pdf · KAPLAN LSAT PREP LSAT RELEASED TEST I EXPLAINED A Guide to the June, 1991 LSAT KAPLAN The answer to the

LSAT PREP __________________________________________________________________ LSAT Test I Explained: Section I

12 © K A P L A N

• A strong mental roadmap of the passage makes inference questions much easier.Knowing where to look is more than half the battle. For example, knowing that the“crucial factors” of the two approaches are discussed in ¶ 4 makes this questionmuch less difficult.

18. (E)Lines 41-43 indicate that the judge is supposed to be an “involved manager” under theinquisitorial system.

(A) and (C) The word passive in “passive observer” (A) and the word uninvolved in“uninvolved administrator” (C) run directly counter to the sentiment expressed in lines 41-43.

(B) and (D) Nothing in the text suggests that the judge under the inquisitorial system issupposed to be “biased” (B) or “aggressive” (D). He or she is simply supposed to manageand guide the legal proceedings.

• If you took note of the passage’s structure, it should have been a snap to go back tolines 41-43 for the answer. If you didn’t note the passage’s structure, you might havewasted valuable time looking for the lines that contain the answer.

19. (B)¶1 indicates that “the victim of a crime fashioned a remedy and administered it privately”under the system of private vengeance. In contrast, under both the adversarial andinquisitorial systems, society is responsible for taking legal action against a criminal—seelines 11-14 and lines 30-36 for the adversarial and inquisitorial systems, respectively.

(A) and (C) The defendant is the person accused of committing the crime, and presumablynever had much of an incentive to initiate legal action. Responsibility for initiating legalaction under the system of private vengeance rested with the victim, not the victimizer.

(D) Again, the victim, not the court system, was responsible for taking legal action underthe system of private vengeance. Indeed, the nature of this system suggests that there wereno courts where it held sway.

(E) Again, under the adversarial and inquisitorial systems, society, not judges, areresponsible for taking legal action against criminals.

• Don’t be intimidated by long question stems—they don’t necessarily indicatedifficult questions.

Page 15: LSAT - dl.keywin.orgdl.keywin.org/a/a/aafd9027c6dbefbc457fbda910414daf.pdf · KAPLAN LSAT PREP LSAT RELEASED TEST I EXPLAINED A Guide to the June, 1991 LSAT KAPLAN The answer to the

LSAT PREP __________________________________________________________________ LSAT Test I Explained: Section I

© K A P L A N 13

20. (D)Under the adversarial system, according to lines 17-19, the defendant is responsible forconducting his own pretrial investigation. In other words, the defendant is responsible formounting his own defense. In contrast, lines 30-36 point out that under the inquisitorialsystem, the public prosecutor has an important role to play in the defense, which conflictswith (D).

(A) According to ¶s 3 and 4, both the prosecution and the defense have a duty to searchfor the facts of a case under the inquisitorial system, which stands in marked contrast to theless cooperative adversarial system.

(B) Lines 30-38 make the point that the inquisitorial system “encourages full disclosure ofevidence.”

(C) Lines 41-43 make the point that the judge plays “an active role in the conduct” of a trialcarried out under the inquisitorial system.

(E) Lines 51-55 suggest that the inquisitorial system “favors the innocent.”

• In all/EXCEPT questions, you’re generally asked to look for the choice that isn’t true.

21. (B)In lines 29-30, the author calls the inquisitorial system “historically superior to theadversarial system” and in line 51 he trumpets the inquisitorial system’s “thoroughness” inascertaining the facts. Thus, choice (B) nicely captures the author’s mood toward theinquisitorial system.

(A), (D), and (E) mistakenly suggest that the author has qualms about the inquisitorialsystem.

(C) The author never claims that the inquisitorial system will replace the adversarialsystem; he simply claims that it’s better.

• If you picked up on the author’s positive attitude toward the inquisitorial system asyou read through the passage, you could have immediately eliminated three of thefive choices in this question, making your task of choosing the correct answer muchsimpler.

Page 16: LSAT - dl.keywin.orgdl.keywin.org/a/a/aafd9027c6dbefbc457fbda910414daf.pdf · KAPLAN LSAT PREP LSAT RELEASED TEST I EXPLAINED A Guide to the June, 1991 LSAT KAPLAN The answer to the

LSAT PREP __________________________________________________________________ LSAT Test I Explained: Section I

14 © K A P L A N

Passage 4—Professions(Q. 22-28)

Topic and Scope: Professions; specifically, the definition of the term profession.

Purpose and Main Idea: The author’s purpose is to define the term profession in order todistinguish it from other types of endeavors. The author’s main idea is that a profession isdistinguished from other endeavors by the commitment shown by its practitioners.

Paragraph Structure: ¶1 introduces the issue of distinguishing between a profession andother endeavors and dips into the history of the term profession. ¶2 and the first part of ¶3discuss definitions of the term that the author rejects—definitions that differentiateprofessions from other endeavors based on learning/knowledge and honor/prestige. Therest of ¶3 and ¶4 provide the author’s definition of the term profession: a profession isdistinguished from other endeavors by the “devotion” of its members to some noble wayof life.

The Big Picture:

• Notice the structure of this passage: ¶1 introduces a “problem”; ¶s 2 and 3 describe“solutions” to the problem that the author rejects; and ¶s 3 and 4 describe theauthor’s “solution” to the problem. On test day, don’t be surprised if you see asimilarly structured passage.

• Nonscience passages sometimes contain complex prose. As in science passages thatcontain complex prose, resist the temptation to understand the passage in detail thefirst time through it. Instead, just try to grasp the passage’s purpose and structure.Only shoot for a finer understanding of the passage if the questions demand it.

The Questions:

22. (E)This choice nicely captures the sentiments expressed by the author in lines 55-67.

(A) and (C) Others have put forward the notion that professions can be distinguished fromother endeavors based on prestige (A) and knowledge (C). The author himself rejects thesecriteria.

(B) In ¶2, the author rejects the notion that “know-how in a particular field” distinguishesprofessionals from non-professionals.

(D) The author never links professional status to the pursuit of “political justice.”

• The correct answer to the first question in a set will often be based on informationnear the end of the passage. Just another reason to read the entire passage.

Page 17: LSAT - dl.keywin.orgdl.keywin.org/a/a/aafd9027c6dbefbc457fbda910414daf.pdf · KAPLAN LSAT PREP LSAT RELEASED TEST I EXPLAINED A Guide to the June, 1991 LSAT KAPLAN The answer to the

LSAT PREP __________________________________________________________________ LSAT Test I Explained: Section I

© K A P L A N 15

23. (D)The author’s main idea is that professionalism requires a commitment to a noble way oflife. Since the author makes this point by concentrating on the case of the medicalprofession, choice (D) is an accurate expression of the author’s main idea.

(A) The etymology of the word profession is a detail that appears in ¶1.

(B) The author never says that it’s wrong to recognize the knowledge and skills ofphysicians; he simply thinks that these things aren’t responsible for medicine’s status as aprofession.

(C) The author acknowledges that the work of physicians is under attack, but never claimsthat this has resulted from widespread misunderstanding. Besides, this acknowledgment isjust a detail in ¶1.

(E) It’s not physicians who think of themselves as “technicians”; it’s others who think ofthem as such. Besides, this, too, is just a detail in ¶1.

• Note that you could have eliminated the four wrong choices simply by recognizingthat they focus on passage details.

• It’s a good idea to read through all of the choices before you endorse any of them.Choices that focus on details may be tempting at first, so it’s important to give all thechoices a shot.

24. (C)The entire passage concerns the nature of professions, using the medical profession as anexample.

(A) Efforts to redefine medicine as a trade are mentioned before lines 7-10, but the passagedoesn’t address the question of whether these efforts are likely to succeed.

(B) The author never claims to dislike “governmental regulation” or “consumer advocacy.”He simply mentions that such things currently pose a challenge to the status of the medicalprofession. Moreover, the author mentions these things prior to posing the question inlines 7-10.

(D) What suggestions? The author never offers suggestions for rallying people to thedefense of physicians, though he personally defends their status.

(E) is beyond the scope of the passage. The author isn’t interested in the origins of words ingeneral; only the origins of the words trade and profession.

• This question demonstrates the importance of following the flow of the author’sargument and noting paragraph topics. With a roadmap of the passage, backtrackingto find the role of lines 7-10 is much easier.

Page 18: LSAT - dl.keywin.orgdl.keywin.org/a/a/aafd9027c6dbefbc457fbda910414daf.pdf · KAPLAN LSAT PREP LSAT RELEASED TEST I EXPLAINED A Guide to the June, 1991 LSAT KAPLAN The answer to the

LSAT PREP __________________________________________________________________ LSAT Test I Explained: Section I

16 © K A P L A N

25. (D)The author notes that trades are based on specific knowledge, but never mentions what thisknowledge is.

(A) How society treats physicians is addressed in lines 1-7.

(B) Lines 51-55 draw an analogy between teaching and medicine.

(C) The public nature of a profession is made amply clear in lines 55-60.

(E) Lines 60-63 distinguish a livelihood from a profession.

• Don’t hesitate to use a process-of-elimination strategy if you have to. Choice (D) is abit tricky: eliminating the other four choices makes it easier to see that it is indeedcorrect.

26. (E)This sentiment is expressed in ¶s 3 and 4.

(A) What new perspective? The author argues that society should continue to recognizephysicians as professionals.

(B) The author never addresses the issue of whether some professionals have beendemoralized by public opinion.

(C) The author mentions governmental regulation of and litigation against the medicalprofession, but doesn’t discuss their effects on the medical profession.

(D) The author never claims that “most professionals have come to be consideredtechnicians.” He only notes efforts to classify physicians as technicians.

• In “attitude” questions that have long answer choices (i.e., choices that are longerthan one or two words), you must read the whole choice. Don’t pick a choice justbecause the first word or two appears to reflect the author’s point of view.

27. (B)Since the passage ends with the author providing his ethics-based definition ofprofessionalism, it’s logical that a follow-on ¶ could begin with an objection to thatdefinition.

(A) The author discusses trades only in ¶1; hence, it’s not likely that he would come backto this issue in a hypothetical fifth ¶.

(C) The role of the “community” is not something that the author discusses; thus, it’s notlogical to think that he’d suddenly take up this issue in a hypothetical fifth ¶.

Page 19: LSAT - dl.keywin.orgdl.keywin.org/a/a/aafd9027c6dbefbc457fbda910414daf.pdf · KAPLAN LSAT PREP LSAT RELEASED TEST I EXPLAINED A Guide to the June, 1991 LSAT KAPLAN The answer to the

LSAT PREP __________________________________________________________________ LSAT Test I Explained: Section I

© K A P L A N 17

(D) In the passage, the author defines the essential nature of the medical profession; so,why would he suddenly express doubts about this issue?

(E) The passage never refers to the “whole body” or the “meaning of illness”; so, it’sunlikely that the author would suddenly introduce these ideas in a follow-on ¶.

• Context is all important in a question like this one. To determine what idea wouldlogically come next, you’ve got to have a sense of what idea the passage closed with.

28. (C)From the end of ¶1 through the middle of ¶3, as we’ve already noted, the author isprimarily concerned with raising and dismissing alternative definitions of the termprofession.

(A) That “something else” is discussed from line 47 onward.

(B) What efforts? The author dismisses alternative definitions of professionalism, but that’snot the same thing as saying that he dismisses “efforts” to redefine the term.

(D) Lines 18-42 do contain a discussion of linguistics and history, but only in order todismiss alternative definitions of professionalism, not to clarify this term’s meaning.

(E) Distinguishing between trades and professions is part of the author’s larger purpose ofrefuting definitions that don’t satisfy him.

• The correct answer to this sort of “mini-global” question must be broad enough tocover the contents of all of the cited lines, not merely a portion of them.

Page 20: LSAT - dl.keywin.orgdl.keywin.org/a/a/aafd9027c6dbefbc457fbda910414daf.pdf · KAPLAN LSAT PREP LSAT RELEASED TEST I EXPLAINED A Guide to the June, 1991 LSAT KAPLAN The answer to the

18 © K A P L A N

SECTION II:

LOGIC GAMES

Page 21: LSAT - dl.keywin.orgdl.keywin.org/a/a/aafd9027c6dbefbc457fbda910414daf.pdf · KAPLAN LSAT PREP LSAT RELEASED TEST I EXPLAINED A Guide to the June, 1991 LSAT KAPLAN The answer to the

LSAT PREP _________________________________________________________________ LSAT Test I Explained: Section II

© K A P L A N 19

GAME 1 — Trade Representatives(Q. 1-7)

The Action: In the second sentence of the opening paragraph, we’re told that there are sixchairs around a “circular table,” a big clue that this is a circle sequencing game. Our hunchis confirmed when the intro goes on to identify our task: placing six trade reps around thatcircular table. Even though circle sequencing games have been absent from recent LSATs,one could show up on your test. Even if you don’t see a game exactly like this one, thetechniques and analytical skills you need to solve this type of game are similar, if notidentical, to those required for any game you’ll face. The Key Issues are:

1) Which representatives can, must, or cannot occupy adjacent chairs?2) Which representatives can, must, or cannot sit between which other representatives?

In the majority of circle sequencing games, another key issue is who is across from whom.How do we know that this game is not concerned with who’s across from whom? A quickscan of the questions tells us. None of the question stems or answer choices include thephrase “across from” or “opposite from.” Therefore, we know that we needn’t concernourselves with this element here.

Another important issue to look for in a circle sequencing game is the left/right issue. Weneed to recognize whether the game is concerned with who sits to the left and right ofwhom around the circle. How can we determine if this is an issue? A quick scan of thegame will tell us. Nowhere in the intro, rules, or questions do the words “left” or “right”occur. As indicated clearly in the rules and questions, the game is concerned only withwho is next to and between whom.

Page 22: LSAT - dl.keywin.orgdl.keywin.org/a/a/aafd9027c6dbefbc457fbda910414daf.pdf · KAPLAN LSAT PREP LSAT RELEASED TEST I EXPLAINED A Guide to the June, 1991 LSAT KAPLAN The answer to the

LSAT PREP _________________________________________________________________ LSAT Test I Explained: Section II

20 © K A P L A N

The Initial Setup: There are two ways to lay out a circle sequencing game. We can eitherdraw a circle (and place the entities in chairs around it) or draw intersecting lines like thespokes of a wheel (and place the entities at the ends of those spokes). The number of spokeswill depend upon the number of entities we’re placing around the table. The “spoke”method is often preferable because it clearly shows not only which chairs are next to eachother, but also which chairs are across from each other. As we mentioned above, mostcircle sequencing games are concerned with who’s across from whom, but scanning thequestions, it’s evident that this game isn’t. From the scan of the questions, it’s also evidentthat even though the game numbers the chairs 1 through 6, none of the questions but thefirst mentions a chair number. So we won’t bother numbering the chairs in our sketch:

K L M N pO

The Rules:

1) Pairs of entities are quite useful. Two entities that must always be together (or adjacentin this game) give us a solid point from which to begin work on each question. Rule 1provides us with P and N as such a pair. Under Key Issues, we already recognized that thisgame isn’t concerned with who’s to the left or right of whom, so we don’t need to take thatinto account when symbolizing the rules. “ALWAYS PN” will serve as a reminder thatthese two must always sit together. (If this game did involve the left/right issue, we’d haveto write “ALWAYS PN OR NP.”)

2) This rule may have looked complex and confusing at first, but remember to ask yourselfwhat this rule means, not just what it says. After interrogating the rule in this way, ithopefully appeared neither complex nor confusing. It simply says that L must always sitnext to M or N or both of them. Again, this game isn’t concerned with left/right, so “LM,LN, or MLN” is a good way to keep this rule in mind.

3) Here we’re given a pair of representatives who can’t sit next to each other. Knowing whocan’t sit together isn’t as useful as knowing who must sit together (as in Rule 1), but thistype of rule does serve to limit the possibilities. K and M must never be next to each otheraround the table. Write “NEVER KM” to serve as a reminder.

4) is an if-then rule, so be sure to take your time and carefully think through the rule andits contrapositive. If O is sitting next to P, then O isn’t also next to M. We can get the

Page 23: LSAT - dl.keywin.orgdl.keywin.org/a/a/aafd9027c6dbefbc457fbda910414daf.pdf · KAPLAN LSAT PREP LSAT RELEASED TEST I EXPLAINED A Guide to the June, 1991 LSAT KAPLAN The answer to the

LSAT PREP _________________________________________________________________ LSAT Test I Explained: Section II

© K A P L A N 21

contrapositive simply by reversing these terms and negating them. If O is sitting next to M,then O isn’t also next to P. “If OP, then no OM” and “If OM, then no OP” should keep therule and its contrapositive in our minds.

Key Deductions: The most important deduction here is recognizing that this game isn’tconcerned with who’s across from whom and who’s to the right or left of whom. Theserealizations may not seem like enormous deductions, but they give us quite a leg up onthis game. Now, we don’t have to waste time worrying about those issues, and this alsoleaves us greater flexibility to toss entities into the circle at the beginning of each question.Furthermore, we can simply throw the P/N pair from Rule 1 into the circle as the startingpoint when there seems to be no other logical place to begin (since we need not worryabout the order they take as long as they’re adjacent).

The Final Visualization: Here’s what we have as we go on to the questions:

K L M N pO

ALWAYS PN

NEVER KM

LM, LN, or MLN

If OP, then no OM

If OM, then no OP

Page 24: LSAT - dl.keywin.orgdl.keywin.org/a/a/aafd9027c6dbefbc457fbda910414daf.pdf · KAPLAN LSAT PREP LSAT RELEASED TEST I EXPLAINED A Guide to the June, 1991 LSAT KAPLAN The answer to the

LSAT PREP _________________________________________________________________ LSAT Test I Explained: Section II

22 © K A P L A N

The Big Picture:

• Time taken up front is especially important in all Logic Games, because a carelessmaster sketch can turn a relatively straightforward game like this into a nightmare. Asketch, first and foremost, should be a tool to help you quickly and accuratelyanswer the questions and accumulate points. One instance of sloppy thinkingduring the setup can foil both of these goals.

• With a sketch as simple as this one, you shouldn’t hesitate to redraw it as needed. Itwould most likely take much more time to figure everything out in your head than itwould to quickly recopy the sketch.

• Not all games are ripe with deductions. However, just because a game doesn’t haveany earth-shattering deductions doesn’t mean that there’s no work to be done in thesetup. Make sure that you understand the game’s action and that you have translatedthe rules into an accurate, accessible form.

• Pay attention to the “scope” of the game. Although this term is usually reserved forLogical Reasoning, it applies here as well: The scope of a game includes the KeyIssues that will govern the game’s action as well as your work with the questions.Anything else is outside the scope of the game, and you needn’t concern yourselfwith such distractions. Here, we saw that the entire action is governed by “next to”and “between” — “across from” and “left and right of” play no part. Recognizingthis, and narrowing your focus accordingly, makes the game that much easier tohandle.

• Sometimes a brief scan of the questions can be very useful, especially if you’restruggling to understand the game’s Key Issues. Here, a quick scan of the questionstells us which aspects of the typical circle sequencing game are important, andwhich are not. Furthermore, if you scanned the questions during the setup stage,you probably noticed that aside from Q. 1, the issue of naming the chairs 1 - 6 islargely irrelevant, which may have saved you time in creating your sketch on thepage.

The Questions:

1. (B)Oh good, here’s an acceptability question. Normally we’d just check each rule against thechoices and eliminate the choices that violate a rule. However, there’s something we needto notice about this acceptability question. Thanks to the circular design of the table, thefirst person of each list (in chair 1) is next to the last (in chair 6). For example, we need torealize that, in choice (A), K sits next to L. With that in mind, we can treat this just like anyother acceptability question; just grab each rule and cross off any choice that violates thatrule. Rule 1 is violated by answer choice (E) which doesn’t have P next to N. Rule 2requires that L be next to M, N, or both. Answer choice (A) has L next to O and K (L in seat6 and K in seat 1) and so can be eliminated. Rule 3 forbids K to be next to M which answerchoice (D) tries (M in seat 6 wraps around and is next to K in seat 1). Finally, the

Page 25: LSAT - dl.keywin.orgdl.keywin.org/a/a/aafd9027c6dbefbc457fbda910414daf.pdf · KAPLAN LSAT PREP LSAT RELEASED TEST I EXPLAINED A Guide to the June, 1991 LSAT KAPLAN The answer to the

LSAT PREP _________________________________________________________________ LSAT Test I Explained: Section II

© K A P L A N 23

conditional Rule 4 is violated by answer choice (C) with has O next to both P and M.Choice (B) remains and is the credited response.

• Acceptability questions are useful tools. First of all, they can give you an easy point.Furthermore, they give you an opportunity to see first-hand how the rules andentities interrelate. Finally, you are given one complete grouping that works (theanswer).

• This question is a great example of the importance of paying attention to non-indented rules—those bits of information often hidden in the opening paragraph.Here we were told that “successively numbered chairs” are adjacent, and chairs 1 and6 are also adjacent. These rules are every bit as important as those of the indentedvariety. Look for them.

2. (A)The stem says that L sits next to P. Get this new information down on the page. The gameisn’t concerned with who’s to the left and right, so just make a new sketch and put L and Pnext to each other. Where to now? Look for rules that have L or P in them. Rule 1 says thatP must sit next to N. L is on one side of P, so N must be on the other. Rule 2 says that Lmust sit next to M or N or both. Here N is on one side of P with L on the other side, so Mmust be next to L. Who’s left? K and O are left to occupy the remaining adjacent chairs inbetween M and N. Rule 3 says that K can’t sit next to M, so K must sit next to N while O sitsnext to M. The complete ordering is now set, so it’s a simple matter of skimming thechoices for the pair that must sit together. K and O must sit next to each other and (A) is theanswer.

• Get any new information down in your test booklet and see where it leads. Look forrules that have the same entities as the new information in the stem. That is howyou’ll find the answer.

• In a typical “must be true” question, the wrong answer choices will be things thateither can, but need not, be true, or things that must be false. Here, that wasn’t thecase. All of the wrong choices had to be false. You knew this because we were able todeduce the complete ordering around the table. This is helpful because knowingwhat the wrong choices will look like allows you to eliminate them more quickly.

Page 26: LSAT - dl.keywin.orgdl.keywin.org/a/a/aafd9027c6dbefbc457fbda910414daf.pdf · KAPLAN LSAT PREP LSAT RELEASED TEST I EXPLAINED A Guide to the June, 1991 LSAT KAPLAN The answer to the

LSAT PREP _________________________________________________________________ LSAT Test I Explained: Section II

24 © K A P L A N

3. (B)Once again, get the new information down and check the rules. We’re not concerned withthe “left/right” issue, so just draw a new sketch and put K between L and P. Now look forrules with K, L, or P. Rule 1 says that P and N must always sit next to each other. K is nextto P on one side, so N must be on the other side of P. Our new sketch clearly shows that Land N aren’t next to each other, which means it’s time to enact Rule 2: L must have Mbeside her. Only one entity and one slot remain, so O will fill in the chair between M andN. The new and complete sketch looks like this:

K

L

M N

p

O

The question asks for the reps that M sits between, and that’s L and O, choice (B).

• Let your pencil work for you. In this question, we’re able to deduce everyone’splacement from the information in the stem. All you had to do was dash out a quickredrawing of the sketch, add the new information, and you were off to the races.However, if you had tried to figure it all out in your head instead of making a newsketch, it would have taken much longer, and you’d be much more likely to make amistake.

Page 27: LSAT - dl.keywin.orgdl.keywin.org/a/a/aafd9027c6dbefbc457fbda910414daf.pdf · KAPLAN LSAT PREP LSAT RELEASED TEST I EXPLAINED A Guide to the June, 1991 LSAT KAPLAN The answer to the

LSAT PREP _________________________________________________________________ LSAT Test I Explained: Section II

© K A P L A N 25

4. (E)Once again, all you have to do is create a new sketch and add the new information: N isnext to M. By now you should be used to putting P and N together. M is on one side of N,so P must be on the other. You should also be used to placing L next to M and/or N.There’s no open chair next to N, so L must go next to M. O and K are left to fill in the twochairs between P and L, and this time nothing forbids either of these people from takingeither of the remaining chairs. The sketch therefore looks like this:

L

MN

p

K/OK/O

The question asks which reps K can sit between. Depending on where K and O sit, K caneither sit between P and O or O and L. The testmakers chose the former in choice (E).

• Once you’ve created a complete and accurate master sketch, wean yourself away fromthe actual rules as printed on the page. By this point in the game, you’re probablyused to always putting P and N together, always putting L next to M and/or N, andalways keeping K and M apart. All of this should be second nature, so why wastetime checking the actual printed rule each time you use it? You symbolized each rulein your master sketch, so you should never have to read the actual rules again. Thekey to this is making sure that your master sketch is accurate. You can’t dependsolely on your master sketch (and various redrawings of it) unless it’s totallyaccurate. One mistake in your setup can cost you valuable points.

5. (E)We’re helped immensely in this question by referring to our previous work. M is on oneside of L, and the question asks for a complete list of all of the reps who could be on theother side. In the previous question, Q. 4, L was next to M on one side, and could be next toeither K or O on the other. This immediately allows us to eliminate (A), (B), and (C), noneof which includes O. (Incidentally, (C) is doubly wrong by not including K.) (D) and (E)are left. How are these two choices different? (E) includes N while (D) does not. If we canprove that N can be on the other side of L, then (E) is the answer; if not, we go with (D). Is itpossible for N to sit on the other side of L? Sure, here is the complete ordering:

Page 28: LSAT - dl.keywin.orgdl.keywin.org/a/a/aafd9027c6dbefbc457fbda910414daf.pdf · KAPLAN LSAT PREP LSAT RELEASED TEST I EXPLAINED A Guide to the June, 1991 LSAT KAPLAN The answer to the

LSAT PREP _________________________________________________________________ LSAT Test I Explained: Section II

26 © K A P L A N

L

M

N

p

K

O

Neri must be included, so (E) gets the point for Q. 5.

• When possible, use your work from previous questions to help eliminate choices inlater questions. Here, without doing any new work, you could eliminate three of thefour wrong answer choices. This is how you’ll have time to get to each question inthe section. A little time saved here and a little time saved there makes quite adifference by the end of the section. And . . .

• Once you’ve narrowed down the answer choices, you can save more time by usinggood answer choice strategies. When the choices contain lists, ask yourself “How dothese remaining choices differ?” If one choice contains an entity not contained in theother remaining choices (like Neri in choice (E) above), you can save time bychecking for that entity.

6. (C)Once again, don’t hesitate to quickly redraw the master sketch including the newinformation from the stem (L next to N). L is on one side of N, so Rule 1 forces P to sit onthe other side of N. Where to now? We’re left with K, M, and O to fill in the remaining threeadjacent slots. We know from Rule 3 that K and M can’t sit next to each other, so they willhave to be separated by O. Either K or M will sit next to L while the other sits next to P—and don’t forget O in between K and M, no matter which way K and M sit. With theexception of K and M, this ordering is completely set. Now move on to the choices,keeping in mind that we’re looking for the choice that must be false.

(A) must be true. K must sit between either O and L or O and P.

(B) could be true. M could sit between L and O with K sitting between P and O.

(C) No way. O can’t sit next to P. She must sit between K and M, as we deduced. (C) mustbe false and is the answer.

(D) must be true. We deduced from the information given in the stem that N must sitbetween L and P.

Page 29: LSAT - dl.keywin.orgdl.keywin.org/a/a/aafd9027c6dbefbc457fbda910414daf.pdf · KAPLAN LSAT PREP LSAT RELEASED TEST I EXPLAINED A Guide to the June, 1991 LSAT KAPLAN The answer to the

LSAT PREP _________________________________________________________________ LSAT Test I Explained: Section II

© K A P L A N 27

(E) must be true. As we saw in our original analysis and again in the context of correctchoice (C), O must sit between K and M.

• By Q. 6 in any game, the rules and deductions you’ve made should be second nature.You should be used to placing P next to N and L next to M and/or N since you’vedone this with every question so far. By this point, strive to be working withconfidence, sure of yourself and your deductions.

7. (E)This stem tells us that K sits next to O. And we can’t take it any further. It looks like we’lljust have to check each choice. But don’t despair! Use your previous work. Look forinstances where K and O sat next to each other and see which reps L was sitting between inthose situations. This question asks who L can’t sit between, so we can eliminate any choicethat is disproved by those previous instances. In the previous question, K was next to O,and in that setup, L could sit between either K and N or M and N. Eliminate choices (B)and (C). Where else was K next to O? In Q. 4, K and O sat together, and in that scenario wesaw it was possible for L to sit between K and M—eliminate choice (A). Finally, in question2, K and O sat together, and L sat between M and P. Eliminate choice (D). Choice (E)remains and is the answer.

• This question gave you a good clue that using your previous work was the way to go.Even with the new information from the stem, you couldn’t make any deductions toget even close to the answer. You had to just check each choice. When you see such aquestion, chances are good that you can use your previous work to eliminate at leastsome of the wrong choices—here, however, we were able to eliminate them all!

Page 30: LSAT - dl.keywin.orgdl.keywin.org/a/a/aafd9027c6dbefbc457fbda910414daf.pdf · KAPLAN LSAT PREP LSAT RELEASED TEST I EXPLAINED A Guide to the June, 1991 LSAT KAPLAN The answer to the

LSAT PREP _________________________________________________________________ LSAT Test I Explained: Section II

28 © K A P L A N

GAME 2 — Computers and Printers(Q. 8-13)

The Action: After previewing the introductory paragraph and rules, we see that this gameasks us to match up four computers and four printers with the year in which each waspurchased—a matching game. The eight machines are in four offices numbered 1 through4, one computer and one printer per office. The Key Issues will deal with:

1) In what year was each computer and printed bought?2) What computer or printer can, must, or cannot have been bought earlier or later thanwhat other computer or printer?3) What computer or printer can, must, or cannot have been bought the same year as whatother computer or printer?

The Initial Setup: One of the most important choices we have to make with this game isdeciding how to organize its information. As with most matching games, you can eitheruse a grid or lists to keep track of the info. Looking at this game, a grid might be slightlymore useful than the lists. Why? Well, look at the information. When deciding how toorganize a game, look for what is variable and what is definitely set. Here, the four officesare definitely set, and one computer and one printer in each are definitely set. The onlything that is variable is the date when each machine was purchased. So we have two fixedpoints and one variable. That is a hint that a grid might be the best way to go. One fixedpoint will go along the top of the grid and one will go along the side. We’ll fill the grid inwith the variable aspect, in this case, the date:

1

2

43

7 8 9

C P

Note that we shortened the dates from 1987, 1988, 1989 to just 7, 8, 9. This is a good time-saving trick. The dates are all identical except for the last digit, so we’ll only concernourselves with that digit.

Page 31: LSAT - dl.keywin.orgdl.keywin.org/a/a/aafd9027c6dbefbc457fbda910414daf.pdf · KAPLAN LSAT PREP LSAT RELEASED TEST I EXPLAINED A Guide to the June, 1991 LSAT KAPLAN The answer to the

LSAT PREP _________________________________________________________________ LSAT Test I Explained: Section II

© K A P L A N 29

The Rules:

1) In each office, the computer was bought either earlier than or the same year as theprinter in that office. In other words, if the computer in 1 was bought in 1988, then theprinter in 1 must have been bought in 1988 or 89. Build this directly into the sketch. Inbetween each office’s C and P, draw a “£.”

2) Here’s information that connects the machines in two different offices. The computer inoffice 2 and the printer in office 1 were bought the same year. Build this directly into thesketch by drawing arrows between the boxes representing these two machines.

3) More inter-office information—the computer in office 3 and the printer in office 4 werebought in the same year. Again, build this directly into your sketch by drawing arrowsconnecting these two squares of the grid.

4) This time we’re given two machines bought in different years: the computer in office 2and the computer in office 3. Build this information directly into the sketch by placing a“X” on the line between the corresponding squares in the grid.

5) gives the only concrete information of the bunch. The computer in office 1 and theprinter in office 3 were both bought in 88. Build this right into the sketch.

Key Deductions: This game has a ton of entities and a ton of rules. Both of these facts arehints that there’s some major deducing to be done here. So let’s get started.

Begin with the concrete info in Rule 5. The computer in office 1 was bought in 88. Rule 1says that each office’s computer was bought earlier than or the same year as that office’sprinter. So the printer in office 1 must have been bought in 88 or 89. Add this to thesketch.

Rule 2 says that the printer in office 1 was bought the same year as the computer in office 2.Since we just saw that the printer in office 1 was bought in 88 or 89, so was the computerin office 2. Add this to the sketch.

From there Rule 1 comes back into play. Since the computer in office 2 was bought in 88 or89, the printer in office 2 must also have been bought in 88 or 89. Add this to the sketch.

Back to Rule 5. Since the printer in office 3 was bought in 88, the computer in office 3(bought earlier than or the same year as the printer—Rule 1) must have been bought in 87or 88. Add this to the sketch.

Since the computer in office 3 was bought in 87 or 88, we can combine this with Rule 3 todeduce that the printer in office 4 must also have been bought in 87 or 88. Add this aswell.

Finally, since the printer in office 4 was bought in 87 or 88, Rule 1 means that the computerin office 4 must also have been bought in 87 or 88. Add this to the sketch.

Page 32: LSAT - dl.keywin.orgdl.keywin.org/a/a/aafd9027c6dbefbc457fbda910414daf.pdf · KAPLAN LSAT PREP LSAT RELEASED TEST I EXPLAINED A Guide to the June, 1991 LSAT KAPLAN The answer to the

LSAT PREP _________________________________________________________________ LSAT Test I Explained: Section II

30 © K A P L A N

The Final Visualization: Here’s that ton of information added to the sketch:

1

2

43

7 8 9

C P

8

8

X8/9

7/8

8/9

8/9

7/8

7/8

££££

££££

££££

££££

The Big Picture:

• This game is the quintessential example of why it’s vital that you take an appropriateamount of time up front to work through all of the possible deductions. Here you’reable to narrow down the possibilities to one or two years for all of the machines. Thiswill GREATLY simplify your work when you hit the questions.

• When you’re figuring out how to best organize the information in a matching game,look for what is variable and what is fixed. The variable aspect is what can change,and the questions will (pardon the expression) invariably ask you about thisvariable aspect. Here, the year is the only thing that can change. The four offices areset. The one computer and one printer in each office are also unmovable. Whenusing a grid, the fixed aspects go along the sides of the grid, and you’ll fill in thesquares of the grid with the variable aspect.

• Some students were hesitant to put in this much time during the setup. Theythought that it would be too much trouble to redraw this master sketch over andover. However, with so much already deduced, you’ll find that you don’t need toredraw very often: Unlike in Game 1, eyeballing the answer from the master sketch ispossible in many cases because we’ve already narrowed down the possibilitiesconsiderably. Indeed, some students found that they only had to redraw the mastersketch for Q. 13 (which contains a rule change).

Page 33: LSAT - dl.keywin.orgdl.keywin.org/a/a/aafd9027c6dbefbc457fbda910414daf.pdf · KAPLAN LSAT PREP LSAT RELEASED TEST I EXPLAINED A Guide to the June, 1991 LSAT KAPLAN The answer to the

LSAT PREP _________________________________________________________________ LSAT Test I Explained: Section II

© K A P L A N 31

The Questions:

8. (B)The printer in office 3 was bought in 88 (Rule 5). The only way for the computer in 3 to bebought earlier is if it was bought in 87. Rule 3 says that the computer in 3 and the printer in4 were bought the same year, so the printer in office 4 must have also been bought in 87.Eliminate (D) and (E). Each office’s computer was bought in the same year as or earlierthan the printer in that office (Rule 1), so since the printer in 4 was bought in 87, the onlyyear that the computer in 4 could have been bought is also 87. Eliminate (C). (A) and (B)are all that’s left. We deduced in the setup that the computer in office 2 must have beenbought in 88 or 89. Cross off (A), which leaves (B), the answer.

• Take your time with early questions and use them to get a good grasp on the game’saction. Usually, early questions are slightly easier; use these questions to make sureyou understand what’s going on.

9. (D)Here’s a could be true question with no new information. Normally this type of questionmight take a lot of time to answer. Here, however, with all the good work we did up front,we should be able to eliminate the wrong answer choices quickly. Just compare eachchoice against our master sketch.

(A) No, the printer in 1 must have been bought in 88 or 89.

(B) No, the computer in 2 must have been bought in 88 or 89.

(C) No, the computer in 3 must have been bought in 87 or 88.

(D) Yes, the printer in 4 could have been bought in 88 (or 87). (D) could be true and is theanswer.

(E) No, the printer in 4 must have been bought in 87 or 88.

• Use the early questions to solidify your hold on a game. Here’s how: Let’s say youdidn’t deduce all of the machines down to one or two possibilities up front. Well,here’s your opportunity to catch up. Since the right answer is a statement thatCOULD be true, each of the wrong choices must contain a statement that CANNOTbe true. By testing choice (A) and proving to yourself that the printer in office 1could NOT have been bought in 87 (wrong choice (A)), you’re forced to deduce thatthe printer in 1 must have been bought in 88 or 89. The same holds true for choices(B), (C), and (E). If you miss deductions up front, all is not lost—you can still pickthem up along the way in the process of working through the early questions.

Page 34: LSAT - dl.keywin.orgdl.keywin.org/a/a/aafd9027c6dbefbc457fbda910414daf.pdf · KAPLAN LSAT PREP LSAT RELEASED TEST I EXPLAINED A Guide to the June, 1991 LSAT KAPLAN The answer to the

LSAT PREP _________________________________________________________________ LSAT Test I Explained: Section II

32 © K A P L A N

10. (A)We want as few machines bought in 87 as possible. Check our master sketch. There are onlythree machines that could have been bought in 87. The computer in 3, the computer in 4,and the printer in 4 all were bought in 87 or 88. The stem asks for the fewest number ofmachines bought in 87, so could all three of these have been bought in 88? Sure. Rule 3 saysthat the computer in 3 was bought the same year as the printer in 4, so both of these couldhave been bought in 88. Rule 1 says that the computer in each office must have been boughtthe same year as or before the printer in that office, so the computer in 4 could have beenbought in 88 as well. All of the other machines couldn’t have been purchased in 87 (see KeyDeductions), so it turns out that we need not have any 87 machines. In other words, thefewest possible number of machines that were bought in 87 is 0, choice (A).

• Don’t assume that you must use your pencil for every question. Sometimes you cananswer a question simply by referencing the master sketch. That’s certainly the casehere. Recognizing when you don’t need to redraw the master sketch can save youvaluable time on Test Day.

11. (B)Remember that the printer in each office was bought either the same year as or later thanthe computer in that office (Rule 1). If the computer in 4 was bought in 88, the printer in 4must also have been bought in 88 (we ruled out 89 as a possibility for the printer in 4 underKey Deductions). We’ve just made a new deduction, so stop and see if that is an answerchoice. It’s not (although we can eliminate (E) which has the printer in 4 bought in 89), sokeep looking.

Rule 3 says that the printer in 4 and the computer in 3 were bought the same year, so thecomputer in 3 must have been bought in 88. Is that a choice? No, but we can cross off (D)which has it bought in 87.

Rule 4 says that the computer in 2 can’t have been purchased the same year as thecomputer in 3, so here the computer in 2 must have been bought in 89 (again check KeyDeductions to see how 87 was eliminated for the computer in 2). Is “computer in 2 boughtin 89” an answer choice? No, but we now know that (C) must be false, so cross it off.

Moving along, Rule 2 says that the computer in 2 and the printer in 1 were bought the sameyear, so the printer in 1 was bought in 89, and finally that’s the answer, choice (B).

• When given new information, follow the chain of deductions. Keep looking forrules and deductions that lead you to the next entity and the next deduction.Somewhere along this chain you’ll encounter the answer. Whenever you make adeduction, though, stop and quickly see if that’s a choice. You don’t want to do morework than is needed. Here that technique wasn’t as helpful as it normally is, becausethe answer turned out to be the last stop in our logical chain. However, in “must betrue” questions, the strategy of checking the choices after making new deductionswill save you time more often than not.

Page 35: LSAT - dl.keywin.orgdl.keywin.org/a/a/aafd9027c6dbefbc457fbda910414daf.pdf · KAPLAN LSAT PREP LSAT RELEASED TEST I EXPLAINED A Guide to the June, 1991 LSAT KAPLAN The answer to the

LSAT PREP _________________________________________________________________ LSAT Test I Explained: Section II

© K A P L A N 33

12. (D)If the computer in 3 was bought in 88, Rule 3 dictates that the printer in 4 was bought in 88as well. Rule 4 (computer in 3 and computer in 2 bought in different years) means that thecomputer in 2 was bought in 89. From there, Rule 2 (computer in 2 and printer in 1 boughtin the same year) insists that the printer in 1 was bought in 89. Also, since the computer in 2was bought in 89, Rule 1 means that the printer in 2 was also bought in 89. The stem asks forwhat could be true. The only machine that still has more than one option is the computerin 4 which could have been bought in 87 or 88. Look first at any choice that includes thecomputer in 4. Sure enough, choice (D), the computer in 4 being bought in 87, is the onlypossible choice.

13. (C)And here’s the one question that absolutely must have a new sketch. Now we’re to considerwhat happens when the opposite of Rule 4 is true. The computer in 2 and the computer in3 now are bought in the same year. Take a moment and create a new master sketchincluding this new information. Look at the original sketch and note what information stillholds. The deduction that the printer in 1 was bought in 88 or 89 still holds, as does Rule 2which means that the computer in 2 was also bought in 88 or 89. The stem says that thecomputers in 2 and 3 were bought in the same year, so the computer in 3 must also havebeen bought in 88 or 89. But wait. The printer in 3 was bought in 88, so Rule 1 means thatthe computer in 3 can only have been bought in 88. This also means that the computer in 2was also bought in 88 (thanks to the new information in the stem). From there, Rule 2 meansthat the printer in 1 was bought in 88 as well. Since the computer in 3 was bought in 88,Rule 3 forces the printer in 4 to have been bought in 88. The only machines that possiblyweren’t bought in 88 are the printer in 2 (88 or 89) and the computer in 4 (87 or 88). Theprinter in 2 is the only machine that could have been bought in 89, choice (C).

• Normally a question that changes or adds a rule takes a lot of time and would be anexcellent candidate for skipping the first time around. However, many studentsfound that they had such a good handle on this game that they were able to make thechange and could almost automatically see how that change affected the numbers.The point is this: Be flexible. If you’ve got such a great handle on a game that youcan make a rule change almost all in your head, then do it.

Page 36: LSAT - dl.keywin.orgdl.keywin.org/a/a/aafd9027c6dbefbc457fbda910414daf.pdf · KAPLAN LSAT PREP LSAT RELEASED TEST I EXPLAINED A Guide to the June, 1991 LSAT KAPLAN The answer to the

LSAT PREP _________________________________________________________________ LSAT Test I Explained: Section II

34 © K A P L A N

GAME 3 — Law Firm Hirings(Q. 14-18)

The Action: After conducting an overview of this game, we see that we’re asked to puteight partners in order based on the year in which they joined the firm—a sequencinggame. Moreover, it’s a “free-floating” sequencing game. We call it this because all of theentities are described relative to each other. In “free-floating” games, the important aspect iswhere each entity is relative to the others—Hodges is before Nader, Owens is after Nader,etc. Notice here that a range—1961 to 1968—is given for the entities. But this is still a “free-floating” game since the entities are only listed before or after each other and not in specificyears. The Key Issues will be:

1) When did each partner join the firm?2) What partners can, must, or cannot have joined the firm before and after what otherpartners?

The Initial Setup: With most “free-floating” sequence games, the most natural way towork is vertically. Put the years along the side and list the partners. Notice that, as in Game2, we’ve shortened the years to just 1 through 8. (You could also have worked horizontallyby listing the years left to right like a timeline. Whichever way you choose, just make surethat your sketch is neat and not misleading.)

G H I J K M N O12345678

The Rules:

A good way to start is to locate a partner or partners who might come at the top of thesketch; that is, anyone who doesn’t seem to have joined the firm after other partners. If wescan the rules, we see that H in Rule 1 and K in Rule 2 both fit that description. Let’stherefore start by putting an “H” and a “K” at the top of the sketch.

Page 37: LSAT - dl.keywin.orgdl.keywin.org/a/a/aafd9027c6dbefbc457fbda910414daf.pdf · KAPLAN LSAT PREP LSAT RELEASED TEST I EXPLAINED A Guide to the June, 1991 LSAT KAPLAN The answer to the

LSAT PREP _________________________________________________________________ LSAT Test I Explained: Section II

© K A P L A N 35

1) H joined before N. From the H at the top of the sketch, add a line down to an N.

2) K joined the firm before J. Next to the H at the top of the sketch, we already have a K.From that K draw a line down to a J.

3) N and J both joined the firm before G. There’s already an N and a J in the sketch, so drawlines down from each of them to a G.

4) gives more information about N. N joined the firm before O. From the N in the sketch,draw another line down to an O.

5) Here’s more information about J. J joined the firm before M. From the J in the sketch,draw another line down to an M.

6) G joined the firm before I. From the G in the sketch, draw a line down to an I.

Key Deductions: The first thing to do is check and make sure that all of the entities areincluded in the master sketch. If one isn’t, that’s a “floater” that can be plugged inanywhere. Fortunately, all of the entities are included in the sketch, so we can dependsolely on our master sketch to answer every question we’ll face.

Before moving on, take a moment and review the sketch. Some things should be obvious.For instance, K joined the firm before G, I joined the firm after N, etc. Also take note of thepossibilities regarding the specific years; what entities could have joined the firm in 1961?What entities could have joined in 68? H and K are the only entities that aren’t below anentity, so either H or K must have joined in 61. I, O, and M are the only partners with noentities below them, so they’re the only ones who could have joined in 68. While thesearen’t earth-shattering deductions, it’s a good idea to take a moment and review what thecompleted sketch can tell you. This sets up the game’s parameters in your mind.

It’s also wise to take a moment and recognize what the sketch can’t tell us. Be careful not toassume any relationship between partners that aren’t connected by lines. For example, I, inour sketch, is slightly lower than M and O. But we can’t assume that M and O joined thefirm before I. We don’t know that. Be very careful when dealing with the two “arms” of thesketch. The “K—J—M” arm is not connected to the “H—N—O” arm. Therefore, we can’tdeduce any relationship between the two. M, for instance, is quite low on the sketch, butsince it’s not connected to the “H—N—O” arm, M could have joined the firm even beforeH. Likewise, O could have joined before K.

Page 38: LSAT - dl.keywin.orgdl.keywin.org/a/a/aafd9027c6dbefbc457fbda910414daf.pdf · KAPLAN LSAT PREP LSAT RELEASED TEST I EXPLAINED A Guide to the June, 1991 LSAT KAPLAN The answer to the

LSAT PREP _________________________________________________________________ LSAT Test I Explained: Section II

36 © K A P L A N

The Final Visualization: Be sure you’ve kept all of your notations neat and easilyaccessible:

G H I J K M N O

G

H

I

J

K

M

N

O

1

2

3

4

5

6

7

8

The Big Picture:

• It’s extremely important to review the section before you dive in. Most studentsfound this game, the third in the section, to be the most straightforward. You want tosearch out the most familiar game and start your section there. You’ll give yourself ajump-start with the points and confidence this game should provide.

• Don’t infer relationships that don’t exist. Don’t think that M must have joined afterH. On the contrary, M could have joined before H, N, and O.

The Questions:

14. (C)In a “free-floating” sequence game like this (when all the entities are in the master sketch),just check each choice against the sketch, using it to figure out the choice that isimpossible.

(A) H (or K) could have joined the firm in 61. Answer choice (A) can be true and isn’t theanswer.

(B) If K joined in 61 and J in 62, then H could have joined in 63. (B) can be true and isn’t theanswer.

Page 39: LSAT - dl.keywin.orgdl.keywin.org/a/a/aafd9027c6dbefbc457fbda910414daf.pdf · KAPLAN LSAT PREP LSAT RELEASED TEST I EXPLAINED A Guide to the June, 1991 LSAT KAPLAN The answer to the

LSAT PREP _________________________________________________________________ LSAT Test I Explained: Section II

© K A P L A N 37

(C) Four entities, H, K, J, and N are all above G in the sketch, so at least four partnersjoined the firm before G. The earliest that G could have joined the firm is 65. Answerchoice (C) can’t be true and is the answer. Quickly, here is why choice (D) and choice (E)can be true.

(D) M could have joined the firm anytime from 63 (K and J joined earlier) to 68.

(E) O could have joined the firm anytime from 63 (H and N joined earlier) to 68.

• Usually a “cannot be true” question with no new information takes quite a long timeto answer, and you may have considered skipping this one the first time around.Here, however, you were able to solely depend on the master sketch to quickly findthe answer.

15. (E)Again, we can solely depend on the sketch. If J joined in 62, exactly one partner joinedearlier (the one that joined in 61). Looking at the sketch, K must have joined the firm earlierthan J, so K joined in 61. That’s all we know for sure. Either H or M could have joined in 63,either H or M or N could have joined in 64, and so on. Check each choice looking for theone that can’t be true.

(A) and (B) Either H or M could have joined in 63 after K (61) and J (62).

(C) Could H have joined in 64? Sure, here is the ordering: K, J, M, H, N, G, I, O.

(D) Could N have joined in 64? Sure, here is the ordering: K, J, H, N, M, G, I, O.

(E) K joined in 61 and J joined in 62. At least H and N joined before O, so the earliest Ocould have joined is in 65. (E) can’t be true and is the answer.

16. (D)We’re asked for the latest year that J could have joined the firm. There are two ways to goabout this: We can put as many partners as possible before J, or we can see which partnersMUST be after J. Since the latter deals with what is DEFINITELY true while the formerdeals with what is POSSIBLE only, the latter is most likely going to be simpler and, moreimportantly, quicker. Looking at the sketch, only G, I, and M are definitely placed below J.Therefore if G, I, and M joined the firm in 66, 67, and 68, the latest that J could have joinedthe firm is 65, choice (D).

• There is often more than one way to tackle a question. Strive to choose the tack thatis the quickest and seems most natural to you. Getting the point is of the primaryimportance, but getting the point as quickly and efficiently as possible is vital ifyou’re going to get to every question on the section.

Page 40: LSAT - dl.keywin.orgdl.keywin.org/a/a/aafd9027c6dbefbc457fbda910414daf.pdf · KAPLAN LSAT PREP LSAT RELEASED TEST I EXPLAINED A Guide to the June, 1991 LSAT KAPLAN The answer to the

LSAT PREP _________________________________________________________________ LSAT Test I Explained: Section II

38 © K A P L A N

17. (B)From the master sketch, we’re able to deduce that either I, M, or O must have joined thefirm last, in 68. This question stem assigns O to 65 and M to 67. Therefore, I must havejoined the firm in 68. Now check the master sketch. H and N joined before O in the 61-64slots. K and J joined before M and also before G. Since H and N must occupy two of thefour 61-64 slots, K and J will take the other two slots which leaves G (which must go AFTERK and J) to take the remaining 66 slot. G and I are the only other entities whose slots can bedetermined, choice (B).

• Never relax your critical thinking skills. The question stem asked for the OTHERentities not counting M and O whose positions can be determined. If you glossedover this fact and counted M and O in your final tally, your work would have beenwasted and you would have missed this point.

18. (D)Here we’re to assume that O joined the firm before M, and we’re looking for the earliestthat M could have joined. Since H and N joined the firm before O, all three of these musthave joined the firm before M. K and J joined the firm before M (as always), so that makes atotal of five partners that must have joined the firm before M. These five will at least takeup the 61-65 slots, so the earliest that M could have joined the firm is 66, choice (D).

• This is the ideal game with which to start your section. It’s probably the easiest gameof the bunch and its action is familiar. By beginning your day here, you easeyourself into the section and gain confidence (and the points!). In addition, moststudents can do this game in well under 8 minutes, so they’ve already got extra timeto use on a later, tougher game.

Page 41: LSAT - dl.keywin.orgdl.keywin.org/a/a/aafd9027c6dbefbc457fbda910414daf.pdf · KAPLAN LSAT PREP LSAT RELEASED TEST I EXPLAINED A Guide to the June, 1991 LSAT KAPLAN The answer to the

LSAT PREP _________________________________________________________________ LSAT Test I Explained: Section II

© K A P L A N 39

GAME 4 — Railway Tickets(Q. 19-24)

The Action: After reading the opening paragraph and rules, we see that this game requiresthat we match up the January and February tickets for three railway lines with the color ofeach—a matching game. The action doesn’t become fully apparent until Rule 1 whichintroduces the concept of the “color” of each set of tickets. Some students were thrown bythe notion of “sets” of tickets in the opening paragraph. They were worried that thisimplied a whole lot of tickets. If we read carefully, though, we know that that’s not aproblem. There is one “set” of tickets per month, as simple as that. The Key Issues are:

1) What color is each of the six sets of tickets?2) What sets of tickets can, must, or cannot be the same color as what other sets of tickets?

The Initial Setup: As with Game 2 and other matching games, a grid or lists both workwell. In Game 2, we talked about identifying the variable and fixed aspects when using agrid. What are the variable and fixed points in this game? The two months, January andFebruary, are fixed. They don’t change. The three railway lines are also fixed, not goinganywhere. What can change from line to line and month to month is the color of the sets oftickets. This is the variable aspect. So when creating our grid, the two fixed points (monthsand lines) will go along the top and side, and we will fill in the grid squares with thevariable (color):

PG R Y

Feb

3

2

1

Jan

Page 42: LSAT - dl.keywin.orgdl.keywin.org/a/a/aafd9027c6dbefbc457fbda910414daf.pdf · KAPLAN LSAT PREP LSAT RELEASED TEST I EXPLAINED A Guide to the June, 1991 LSAT KAPLAN The answer to the

LSAT PREP _________________________________________________________________ LSAT Test I Explained: Section II

40 © K A P L A N

The Rules:

1) gives the information that we’ve already included in the sketch above, namely that eachset of tickets will be green, purple, red, or yellow.

2) Let’s take our time and make sure that we fully understand this rule. The January set oftickets of each line (1, 2, and 3) will be a different color from the February set of tickets ofthat line. Let’s do a “what if” to make sure we have a handle on this rule. If the Jan 1(January line 1) set of tickets is red, then the Feb 1 (February line 1) set of tickets cannot bered. Draw a big “X” between each line’s January and February squares in our grid.

3) Again, we need to take our time when decoding this rule. Within each month, the set oftickets of each line must be a different color than the set of tickets of the other two lines.Let’s do another “what if.” If the Jan 1 (January line 1) set of tickets is red, then the Jan 2(January line 2) set of tickets can’t be red. Between each month’s 1 and 2 squares draw a big“X,” and between each month’s 2 and 3 squares draw a big “X.”

4) Of the three sets of January tickets, exactly one must be red. Build this directly into ourgrid. Write “ex. 1 R” at the bottom of the January column.

5) Once more, take care when translating Rule 5. This rule is quite similar to Rule 4. InRule 4, exactly one of the sets of January tickets (either line 1, 2, or 3) was red. Here, exactlyone of the sets of line 3 tickets (either January or February) must be green. At the end of theline 3 row, write “ex. 1 G.”

6) Here is a nice, concrete piece of information. The Jan 2 set of tickets is purple. Build thisright into the master sketch.

7) Finally, Rule 7 states that none of the sets of February tickets are purple. Write “NO P”in each of the February squares in our grid.

Key Deductions: Rule 6 said that Jan 2 is purple. Rule 3 said that within each month, thesets of tickets for the three lines must be of different colors. So we can deduce that Jan 1and Jan 3 can’t be purple. Rule 7 already said that NO sets of February tickets are purple,so we definitely know that Jan 2 is the only set of tickets that is purple.

We can deduce something else from Rule 7 which says that none of the February tickets arepurple. Rule 3 says that each month’s set of tickets must be different colors. Since none ofthe February tickets are purple, that means that all three of the other colors must be usedexactly once for the three sets of February tickets, one green, one red, and one yellow.

Page 43: LSAT - dl.keywin.orgdl.keywin.org/a/a/aafd9027c6dbefbc457fbda910414daf.pdf · KAPLAN LSAT PREP LSAT RELEASED TEST I EXPLAINED A Guide to the June, 1991 LSAT KAPLAN The answer to the

LSAT PREP _________________________________________________________________ LSAT Test I Explained: Section II

© K A P L A N 41

The Final Visualization: Here’s our master sketch:

PG R Y

Feb

3

2

1

Jan

X

XX

X

XX

XP

no P

ex. 1R

ex. 1G

no P

no P

no Pno P

The Big Picture:

• Usually, from the introductory paragraph alone, you’re able to get a handle on agame’s action and create an initial setup. If you’re having trouble doing that,sometimes you need more information found in the rules. In these cases, don’thesitate to read the first few rules to see if they can help cement the action. Here theaction didn’t become apparent until Rule 1.

• Organize the information in a way that’s easiest for you. We used a grid here, butmany people prefer to use lists. As long as you organize the information in a waythat is neat, accurate, and readily accessible, the form you choose is up to you.

The Questions:

19. (E)Rule 5 says that one of the sets of line 3 tickets (either January or February) must be green.The stem says that Jan 3 is red, so Feb 3 must be green, choice (E).

• After a time-consuming setup, don’t be surprised if the first question is fairlysimple. The testmakers like to reward you for taking an appropriate amount of timeup front to reason out the rules and organize the data.

20. (A)What must be true if one of the sets of line 2 tickets is green? Jan 2 is purple (Rule 6), so theonly 2 line left to be green is Feb 2. Is Feb 2 green an answer choice? Nope, keep going.Rule 3 says that no two sets of tickets in the same month can be the same color. Since Feb 2is green, Feb 3 can’t be green. Rule 5 (either Jan 3 or Feb 3 must be green) comes into effectagain and Jan 3 must be green. Is Jan 3 green a choice? No, so keep looking. Rule 4 says that

Page 44: LSAT - dl.keywin.orgdl.keywin.org/a/a/aafd9027c6dbefbc457fbda910414daf.pdf · KAPLAN LSAT PREP LSAT RELEASED TEST I EXPLAINED A Guide to the June, 1991 LSAT KAPLAN The answer to the

LSAT PREP _________________________________________________________________ LSAT Test I Explained: Section II

42 © K A P L A N

one of the January tickets is red, and since Jan 2 is purple and Jan 3 is green, Jan 1 must bered. And that’s choice (A) and the answer.

21. (A)There’s not much to do with this “could be true” question with no new information but tryout each choice.

(A) Could none of the January tickets be green? If none are, we’re left with purple, red,and yellow as the colors of the three sets of January tickets. Jan 2 is purple (Rule 6). Jan 1could be red, and as long as Feb 3 is green (to fulfill Rule 5), Jan 3 could be yellow. Answerchoice (A) could be true and is the answer. Quickly here is why the remaining choices can’tbe true:

(B) Since none of the sets of February tickets are purple (Rule 7), exactly one set of theFebruary tickets MUST be green.

(C) The Jan 2 tickets are purple (Rule 6) and one set of the January tickets is red (Rule 4), soeither Jan 1 or Jan 3 must be red.

(D) Jan 2 is purple, so if one set of January tickets was green and the other set of Januarytickets was yellow, there wouldn’t be any set of January tickets that was red which wouldviolate Rule 4.

(E) Jan 2 is purple, and Rule 7 says that absolutely none of the sets of February tickets canbe purple.

• Negative choices like (A) and (B) are a little slippery. You have to see if it could betrue that something doesn’t happen. Compare this with positive choices like (C), (D),and (E). It’s simpler to check whether or not something positive could be true. Forthis reason, you may have been better served by skipping Q. 21 in favor of Qs. 22 and23 which have all positive answer choices (especially Q. 23 with its positive, shortanswer choices).

22. (B)Another “could be true” question with no new information (but with positive choices, atleast—see Bullet Point to Q. 21 above). Again, not much to do with this one but try outeach choice.

(A) If Jan 1 and Feb 2 are both green, Rule 3 means that none of the remaining sets of ticketscan be green. But Rule 5 insists that either Jan 3 or Feb 3 be green. (A) can’t be true and isn’tthe answer.

(B) Can Jan 1 and Feb 2 both be yellow? Sure: Jan 1—Y, Jan 2—P, Jan 3—R; Feb 1—R, Feb2—Y, Feb 3—G. (B) can be true and is the answer. Quickly, here is why the remainingchoices can’t be true:

Page 45: LSAT - dl.keywin.orgdl.keywin.org/a/a/aafd9027c6dbefbc457fbda910414daf.pdf · KAPLAN LSAT PREP LSAT RELEASED TEST I EXPLAINED A Guide to the June, 1991 LSAT KAPLAN The answer to the

LSAT PREP _________________________________________________________________ LSAT Test I Explained: Section II

© K A P L A N 43

(C) If Jan 1 is yellow, the only set of January tickets left to be red (as Rule 4 requires) is Jan3. Rule 5 then requires that Feb 3 be green, not yellow.

(D) If Jan 1 is green, Rule 3 means that none of the other sets of January tickets are green, soRule 5 again requires that Feb 3 be green, not red.

(E) If Jan 3 is yellow, the only set of January tickets left to be red (as Rule 4 requires) is Jan1. Since Jan 1 is red, Rule 2 doesn’t permit Feb 1 to also be red.

23. (E)If Feb 3 is yellow, Rule 5 forces Jan 3 to be green. Since Jan 3 is green, Jan 1 is left to be red(as Rule 4 requires). Since none of the sets of February tickets are purple, Feb 1 and Feb 2must be green and red. Since Jan 1 is red, Rule 2 means that Feb 1 can’t be red and must begreen. Feb 2 is left to be red. The entire setup is fixed: Jan 1—R, Jan 2—P, Jan 3—G; Feb 1—G, Feb 2—R, Feb 3—Y. The only choice that doesn’t agree with this setup is (E)—only one ofthe sets of tickets is yellow.

• Just by reading the question stem, you should have recognized that this questionwould probably lead to all colors being set for every line’s tickets. Why? The phrase“must be true EXCEPT” was your big clue. In order to come up with four wronganswer choices full of must be trues, you can bet that you’ll need to keep looking fordeductions until you’ve got a near complete setup, if not a totally complete setup.

24. (C)For this question, none of the sets of tickets are purple. This doesn’t affect the sets ofFebruary tickets since none were purple anyway (Rule 7). But since none of the sets ofJanuary tickets are purple (sorry Jan 2, you’re not purple anymore), one will be green, onered, and one yellow. These three colors are the only options for both month’s tickets. Nowcheck the choices looking for the one that can be true.

(A) and (B) We can kill both of these choices immediately. Since none of the sets of ticketsare purple, all of the other colors must be used exactly once per month. Exactly one set ofJanuary tickets and exactly one set of February tickets must be green.

(C) Could neither of the sets of line 2 tickets be green? Sure, here is the complete setup: Jan1—Y, Jan 2—R, Jan 3—G; Feb 1—G, Feb 2—Y, Feb 3—R. (C) could be true and is the answer.

(D) If none of the sets of line 1 or line 2 tickets are yellow, then both Jan 3 and Feb 3 must beyellow which Rule 2 forbids.

(E) If none of the sets of line 2 or line 3 tickets are red, then both Jan 1 and Feb 1 must bered which Rule 2 forbids.

• In Q. 21, you saw two negative choices. In this question, all five choices are phrasedin negative terms. If you were running short on time at the end of a section, Q. 23with its positive choices would be a much better place to look for a point than Q. 24(or Q. 21) with its negative choices.

Page 46: LSAT - dl.keywin.orgdl.keywin.org/a/a/aafd9027c6dbefbc457fbda910414daf.pdf · KAPLAN LSAT PREP LSAT RELEASED TEST I EXPLAINED A Guide to the June, 1991 LSAT KAPLAN The answer to the

44 © K A P L A N

SECTION III:

LOGICAL REASONING

Page 47: LSAT - dl.keywin.orgdl.keywin.org/a/a/aafd9027c6dbefbc457fbda910414daf.pdf · KAPLAN LSAT PREP LSAT RELEASED TEST I EXPLAINED A Guide to the June, 1991 LSAT KAPLAN The answer to the

LSAT PREP ________________________________________________________________ LSAT Test I Explained: Section III

© K A P L A N 45

1. (A)It’s tough to pre-phrase an answer to this one; you simply need to attack the choices insearch of an explanation as to why packing a wound with sugar would so successfully killthose hardy and ubiquitous bacteria. And you needn’t search long: If sugar makes awound dry and if bacteria like moisture, as (A) states, then it’s not surprising that a sugarenvironment tends to spell R.I.P. for bacteria.

(B) So sugar is available for use. But how would one use it? Why would it be successful?

(C)’s phenomenon creates a paradox, not resolves one. We’re trying to figure out why sugarin a wound kills bacteria, not how it might provide an environment in which they thrive.

(D) deals with foods that contain sugar, which is far removed from the treatment described.And in any case, if sugar were to weaken an antibiotic (assuming that such a drug wereprescribed for a wound), that would be harmful rather than helpful. Both (C) and (D) giveus more or less the opposite of what we want.

(E) is unnecessary background information. What does the long-time use of sugar onwounds have to do with why it has the effect it does?

• When trying to strengthen or justify an argument or process, beware of answerchoices (such as (C) and (D), here) that tend to do the opposite of what you seek. Wecall such choices “au contraire” and they are common on the LSAT.

2. (B)This parallel reasoning stimulus can be easily translated in algebraic terms: If color-blind(If X), then can’t differentiate green and brown (then Y). Gerald can’t differentiate thosecolors (Since Y), so he must be color-blind (therefore X). This is a common logic error; it’scalled the fallacy of affirming the consequent—you’re not allowed to simply transpose the ifand then clauses of a conditional statement. (B) is set up in the same way: X is “sufferingfrom sinusitis,” Y is “losing one’s sense of smell,” and Mary plays the role of Gerald.

(A) and (D) represent a straightforward “If X, then Y; William (or Arthur, depending) is X,therefore he’s Y.”

(C) is proper logic. Those who have had jaundice X can’t give blood Y, and so—here comesthe contrapositive—since Jean gave blood not Y, she can’t have had jaundice not X. That’sabsolutely valid and not parallel to the original.

(E) “Since X, therefore Y; Frieda’s X, therefore Z.” Where’d that Z term come from? (E)’s notparallel either.

• When a Parallel Logic stimulus is written in strict formal terms as this one is, youcan easily and carefully translate it into X/Y or A/B terms, translate the five choices,and find the one and only choice that matches.

Page 48: LSAT - dl.keywin.orgdl.keywin.org/a/a/aafd9027c6dbefbc457fbda910414daf.pdf · KAPLAN LSAT PREP LSAT RELEASED TEST I EXPLAINED A Guide to the June, 1991 LSAT KAPLAN The answer to the

LSAT PREP ________________________________________________________________ LSAT Test I Explained: Section III

46 © K A P L A N

3. (E)Wegener’s continental drift theory is widely accepted, we’re told, despite the fact that wedon’t know why the continents move. We do however know that they move—the motionhas been confirmed—and that is evidently good enough for the scientific community. Eachelement of (E) matches up to the passage: The “theory” is Wegener’s; the events postulatedare continental movements; and the explanation for same is what the author has told us ismissing.

(A) illustrates why science might accept a single theory that defines all of nature, but sucha theory is not Wegener’s. (A) fails to explain why this one particular theory has caught ondespite the absence of some key evidence.

(B) The “mathematical nature” stuff has no counterpart in the passage, nor does the issueof science’s overall accuracy.

(C) Well, the one measuring improvement we hear about—the one that allows science toconfirm the movement of continents—has made it easier, not harder, for Wegener’s theoryto take hold.

(D) evokes an irrelevant distinction between single events vs. mass behavior, one that hasnothing to do with the topic and scope of the stimulus discussion.

• This is the LSAT’s very first “principle” question—the now common type in whichwe have to match up a concrete situation to an abstract principle or statute, oroccasionally, vice versa. With such questions, you must pay careful attention to topicand scope—you need to match up each key element of the situation to the statementsin the choices.

4. (D)According to the author, the basis of the deterrence theory is that fear of retaliation keeps(deters) a potential aggressor from attacking. She further argues that maintaining deterrencerequires maximum fear—that is, a potential aggressor must be so convinced of theinvincibility of its potential opponent that it will lay low. From that, (D) must follow: Apowerful nation seeking deterrence will want to let would-be aggressors know just howstrong it really is; that knowledge is what makes deterrence work.

It’s unlikely that you pre-phrased (D); what’s more likely is that after studying thestimulus, you proceeded to examine the five choices and (with luck) chose the right one.Here’s why the others should have been bypassed:

(A)’s necessary condition—the “certain knowledge” of total destruction, signaled by “onlyif”—is not something to which the author is committed. Deterrence simply requires enoughawareness of an opponent’s strength to make the potential aggressor think twice. (A) is tooextreme to be a proper inference.

Page 49: LSAT - dl.keywin.orgdl.keywin.org/a/a/aafd9027c6dbefbc457fbda910414daf.pdf · KAPLAN LSAT PREP LSAT RELEASED TEST I EXPLAINED A Guide to the June, 1991 LSAT KAPLAN The answer to the

LSAT PREP ________________________________________________________________ LSAT Test I Explained: Section III

© K A P L A N 47

(B) offers a case in which military deterrence would not be in effect, because the aggressornation believes itself to be mightier than its potential target. This doesn't mean such anation will definitely attack the other nation for this reason alone, but we certainly can'tinfer, as (B) does, that this country won't attack.

(C) Oh yeah? An equally likely reason that one nation fails to attack another is that it (thefirst nation) simply isn’t warlike and has no designs on the other. (C) as written—implyingthat all peace is based only on fear of retaliation—is deeply cynical at best, loony at worst.

(E) makes a classic LSAT scope shift. The stimulus says that deterrence relies on aggressorsbelieving that a nation has superior retaliatory power. (E) changes this to read that thedefending nation must actually possess that power, a subtle but very important difference.

• Pre-phrasing, as a technique, is less useful in inference questions than in othertypes.

• When attacking inference questions, avoid answer choices, such as (A) here, that aretoo extreme.

5. (C)The conclusion that we’re directed to make Clay “reconsider” is signaled by the phrase“This indicates that...,” and the gist of it is that it’s a rebuttal: For Clay, life as we know itexists nowhere but on Earth, despite the astronomer’s prediction that lots of life is to befound on planets Out There.

Notice that Clay spends most of his letter describing the astronomer’s thought process. Heprovides only one piece of counter evidence, and it’s this: No single planet outside of ourown solar system has been found. On that absence of evidence hangs Clay’s stubborn,categorical insistence that We Are Alone. Well, he’d have to back off if, as (C) states, thereason that no other planets have been found is not that they don’t exist, but that we don’tyet have the means to find them. If (C) is true, then until such time as adequate means ofdetection is developed, the astronomer’s prediction is unscathed and will have to stand.

(A) sounds like a capitulation on Clay’s part, but it’s really outside the scope. The point atissue between Clay and the astronomer is whether life as we know it exists elsewhere thanEarth. Clay could concede the existence of (A)’s weird life forms and still argue that theastronomer is wrong.

(B) Planets and a star like the Sun are, to the astronomer, necessary for life as we know it toexist. That some stars may be planet-less has no effect on either the astronomer’s logic orClay’s.

(D) is something that Clay already accepts as true—his whole rebuttal hinges on his beliefthat it’s the astronomer’s faulty assumption (about other planets) that renders theprediction worthless. Since (D) is in synch with Clay’s reasoning, it cannot serve to makehim reconsider that reasoning.

Page 50: LSAT - dl.keywin.orgdl.keywin.org/a/a/aafd9027c6dbefbc457fbda910414daf.pdf · KAPLAN LSAT PREP LSAT RELEASED TEST I EXPLAINED A Guide to the June, 1991 LSAT KAPLAN The answer to the

LSAT PREP ________________________________________________________________ LSAT Test I Explained: Section III

48 © K A P L A N

(E) is irrelevant. So communication between us and E.T. would be tough, eh. Would it beimpossible? What has this to do with whether life as we know it does or does not exist?

• Not everything in a stimulus argument is germane to the logic. Here, as noted above,Clay Moltz’s argument is confined to the last two sentences; the rest of his letter isbackground information and filler. Always endeavor to pull the evidence andconclusion out, and paraphrase them, for maximum comprehension.

6. (A)The conclusion is that since a necessary condition of Max’s guilt (the unwillingness to askthe cops to investigate) is not met—in fact Max invites investigation—then he cannot beguilty. In the same way, “my” inability to see Lucille in (A) is necessary in order for her tobe in the next room; since “I” haven’t met this necessary condition (i.e. I can see her), thenshe must not be in that room. Yes, it’s simply classic if-then/contrapositive logic.

(B), (C) Each flips (transposes) its if-then statement: If rich, then no Alaskan vacation; sinceno Alaskan vacation, therefore rich (B). If over 40, then no interest in learning; since nointerest in learning, therefore over 40 (C). The stimulus contraposes, rather than transposes,its conditions, so neither (B) nor (C) is parallel.

(D) negates its if-then statement: If good cook, then no cinnamon added; since not a goodcook, cinnamon added. Again, to contrapose—which is what the stimulus and (A) do—means to negate and transpose simultaneously.

(E) is straightforward enough: If one is sociable, then one won’t avoid her friends; Sally’ssociable, therefore she’s not one to avoid her friends. This would work if it concluded“Since Sally avoids her friends, she must not be sociable.”

• Having trouble with the concept of the contrapositive? Just study this question andcorrect choice (A) from time to time. Each of them has it exactly right.

• Use these wrong choices as illustrations of the two most common fallacies: (B) and(C) affirm the consequent, whereas (D) denies the antecedent. Both are commonly-tested.

7. (A)If time is tight—or you find that you cannot follow the argument as presented—this one ispractically screaming, “Skip me for now; come back to me later on.” Handling it requires afirm grasp on the comparisons offered, as well as on the direct relationship that is theauthor’s conclusion. Let’s start with that. According to the author, one will find a higherpercentage of older male ducks when there is a great disparity in sex ratios overall (i.e. allmales : all females). How come? Because among older ducks there are a lot more males,whereas among younger ducks the male : female ratio is much closer to 1 : 1. In otherwords, the more the male ducks strongly outnumber the females, the likelier it is that theolder males will outnumber the younger ones.

Page 51: LSAT - dl.keywin.orgdl.keywin.org/a/a/aafd9027c6dbefbc457fbda910414daf.pdf · KAPLAN LSAT PREP LSAT RELEASED TEST I EXPLAINED A Guide to the June, 1991 LSAT KAPLAN The answer to the

LSAT PREP ________________________________________________________________ LSAT Test I Explained: Section III

© K A P L A N 49

Compare the male : female ratio at the two lakes in question. The western lake’s is close to 1: 1 (55 : 45), while the eastern lake’s is almost 2 : 1 (65 : 35). Which lake, according to theconclusion, should have a greater percentage of older males? The eastern—that’s the lakewith the greater male : female disparity. So the western lake should have a lower percentageof older males . . . and that’s just what (A) points out.

(B) The data in the argument compare the ages of adult and nonadult males. We cannotdraw (B)’s conclusion about adult vs. nonadult ducks—males and females combined.

(C) No conclusion about a total population can be drawn from a male : female ratio.Contrary to (C), the western lake could have many more male ducks than the eastern, solong as the western’s population reduces to 55 : 45 and the eastern’s to 65 : 35.

(D) introduces an issue—hatching time—that is totally removed from the argument. Hence,(D) is completely unacceptable as an inference.

(E)’s conclusion about the age ratio of adult females : nonadult females is unjustified,because no data comparing those groups are provided. The conclusion only concernsmales.

• Remember the high “truth threshold” that an LSAT inference must meet. It’s got tobe something that is certain, based on the text. Don’t settle for anything less!

• If a question strikes you as complex, don’t just bite the bullet and stop to work it allout, no matter what. Be ready and willing to skip it happily and immediately, eager tomove ahead to more gettable points. Let’s face it, if you’re going to leave questionsundone, better they be really tough questions than easier ones. And in any case,even really tough ones will look easier toward the end of the 35 minutes, whenyou’ve already amassed a whole lot of points.

8. (B)There’s a huge scope shift here—did you see it? This author argues that Yellow Dye No. 5should be permitted because its health benefits outweigh its health risks. But the onlybenefit cited is that the yellow color makes people enjoy lemon soda more. That’s a matterof personal preference only! Yet the risk cited—the potential allergic reaction—is a realhealth hazard. By the author’s own logic, any decision on a ban must be based on thecomparison of health benefits and health risks, and so the author has used inappropriateevidence to make her recommendation. That’s the essence of (B).

(A) is flat wrong. The author flatly states that such a risk does exist.

(C)’s assertion that most people may be harmed by additives in general is irrelevant to thediscussion of this particular additive and its benefit/risk ratio.

(D) The benefit/risk ratio that the author recommends evaluating doesn’t hinge on risksbeing proven, so (D) cannot explain the argument’s weakness.

Page 52: LSAT - dl.keywin.orgdl.keywin.org/a/a/aafd9027c6dbefbc457fbda910414daf.pdf · KAPLAN LSAT PREP LSAT RELEASED TEST I EXPLAINED A Guide to the June, 1991 LSAT KAPLAN The answer to the

LSAT PREP ________________________________________________________________ LSAT Test I Explained: Section III

50 © K A P L A N

(E) Consumer awareness is way outside the scope of the argument which concerns thestandards for determining whether or not an additive should be banned.

• To attack an argument, one must stay within the scope of the argument as its authorhas laid it out. Note how most of the incorrect choices depart markedly from thescope here. Departing from the argument’s scope is a sure sign of a wrong answer.

9. (A)The conclusion that we are to weaken is a prediction: that businesses at risk of sufferingenvironmental accidents will now install safeguards, on the seemingly sensible rationalethat it’s now more expensive to pay a fine than to prevent an accident in the first place. Butthat assumes, of course, that those businesses are prudent rather than gamblers by nature. If(A) is true—if businesses typically do not acknowledge the risk of accident—then whywould they start installing safeguards as the author predicts? They’d be far more likely todo nothing and take their chances. (A) severs the connection between evidence andconclusion pretty effectively.

(B) is yet another irrelevant distinction—we’ve seen so many in the section so far, andindeed many Logical Reasoning wrong answers feature such. The argument makes nomention of long- vs. short-term business issues.

(C), if true, leaves us wondering whether the argument is damaged or strengthened. If, as(C) says, making good business sense is a necessary condition of business’s doing the rightthing, environmentally speaking, we’re left to ask the question, “Does planning ahead forpotential disaster make good business sense or doesn’t it?” If Yes, the argument isstrengthened; if No, it’s weakened.

(D) is implying that businesses can write off a fine as if it’s nothing, and so they wouldn’tbother with risk prevention. However, (D) still leaves open the possibility that a fine, evenseen as an “ordinary business expense,” might be so much bigger than the cost ofprevention that the author’s prediction would remain in effect.

(E) Business self-promotion and public awareness—however relevant to the general issuesraised—are irrelevant to the conclusion drawn.

• Remember, we can only strengthen or weaken an argument on the arguer’s ownterms. Choices like (B) and (E), which have nothing to do with those terms, cannot becorrect.

10. (C)This one has a weird question stem. “Necessary to rely on” is LSAT code for “a necessaryassumption.” What they’re getting at is that four of the choices are assumptions necessaryto the reasoning; it follows that the right answer must either be outside the scope, orcontradict the text.

Page 53: LSAT - dl.keywin.orgdl.keywin.org/a/a/aafd9027c6dbefbc457fbda910414daf.pdf · KAPLAN LSAT PREP LSAT RELEASED TEST I EXPLAINED A Guide to the June, 1991 LSAT KAPLAN The answer to the

LSAT PREP ________________________________________________________________ LSAT Test I Explained: Section III

© K A P L A N 51

The issue is this: How freely should advanced technology be passed around? Even ademocracy, the author asserts, must restrict its dissemination when national security orcommercial value is at stake and prohibit access to potential enemies or competitors. Doesthat seem to contradict the last sentence—that free exchange of scientific information isrequired? No: There is a scope difference between scientific information in general, whichthe author argues should be kept free, and advanced technology, which may be restricted.

Now, all of that depends on the ability to make a distinction between that which should befreely exchanged and that which may be prohibited, so (D) is an assumption that the authormust make: There has to be a way to separate out the sensitive tech stuff. By the same token,if potential enemies and competitors are to be frozen out, it has to be possible to identify thepotential foes . . . so (A), too, is an assumption necessary to the logic. If either (A) or (D) isin doubt (here’s the Kaplan Denial Test at work), then one could not follow the author’sdictates with confidence.

Given their references to things like public policy and multinational corporations, theother two necessary assumptions (a/k/a wrong choices) might not seem within the author’sscope, but they sure prove to be so. The author must rely on (B), that the people in ademocracy don’t need to know all about its advanced technology in order for the countryto function. Why? Because if (B) is false—if the people do need that knowledge—then therestrictions on disseminating advanced technology are utterly inappropriate. Similarly, if(E) is false—if one cannot keep advanced technology from crossing borders more or lesswilly-nilly—then the whole idea of keeping such technology out of the hands of selectedcountries is a ludicrous one. The author must believe, with (E), that multinationalbusinesses notwithstanding, a nation can restrict where its advanced technology is shared.

All of the above is moot, of course, if you recognize that (C)’s issue—namely, which type ofpolitical system has more advanced technology—is totally outside the scope of theauthor’s recommendations, which inferably apply to all countries, “even in a democracy.”(That’s what “even” means in this context: The advice is not just restricted to democracies.)With luck you saw (C)’s irrelevant distinction before spending an unwarranted amount oftime on the other four choices.

• When a stem indicates that the right answer is an “odd man out,” consider lookingfor it directly as your first line of attack. Slogging through the four wrong choicesmay take more time. Don’t do so unless you have to.

11. (B)The question wants an inference about typical ice ages, which are mentioned only in thelast sentence, so start there. During an ice age, “however,” precipitation is trapped as ice onice caps. What’s that “however” in contrast to?. It’s the previous sentence, which asserts that“this phenomenon” usually has no effect on the ocean’s composition. Therefore, we can besure that the trapping of precip on ice caps during an ice age does have an effect on theocean’s composition. See how that works? Go back over that again—follow the logic. Now,to understand the effect, we have to understand the cause, the “phenomenon.” To whatphenomenon are they referring?

Page 54: LSAT - dl.keywin.orgdl.keywin.org/a/a/aafd9027c6dbefbc457fbda910414daf.pdf · KAPLAN LSAT PREP LSAT RELEASED TEST I EXPLAINED A Guide to the June, 1991 LSAT KAPLAN The answer to the

LSAT PREP ________________________________________________________________ LSAT Test I Explained: Section III

52 © K A P L A N

It’s in sentence one. Water vapor evaporates from the oceans with more oxygen-16 and lessoxygen-18 than seawater contains. And that should allow you to follow the steps through.If all of that water vapor returned to the oceans as precipitation, then there’d be noproblem; the proportions of oxygen-16 and -18 would remain pretty much stable, just assentence two says they do. But if during an ice age, the water vapor doesn’t return to theoceans—if it’s captured by ice—what’s going to happen to the oceans? Seawater is going tohave less oxygen-16 (because that water vapor not returning to the oceans is oxygen-16-rich), and will have more oxygen-18 (because water vapor is oxygen-18-poor). That secondoutcome is choice (B).

(A) contradicts the text which says water vapor and seawater have manifestly differentproportions of oxygen -16 and -18.

(C) Even setting aside the fact that the term “interglacial periods” never appears in thestimulus—and hence is a poor prospect for an LSAT inference—(C) has it all bollixed up.The issue is how much oxygen-16 and -18 are returned to the oceans, not how muchevaporates from them. There’s no indication that an ice age has any particular impact on thecomposition of the evaporated water vapor.

(D) contains an irrelevant comparison, and in any case precip over land is outside thescope. All of the author’s interests lie at sea. Me hearties.

(E) may be tempting if you start picturing those icecaps melting slowly. But the author’ssole interest is in comparing the composition of normal seawater and ice-age seawater, andthe rate of change is never taken up. For all we know, seawater’s composition changes at thevery same rate whether there’s an ice age or no.

• Don’t just rush through your reading of each question stem. Milk it for all possiblehelp, all possible hints. Here, it’s not enough to recognize Q. 11 as an inferencequestion—you’re told it’s an inference about ice ages, which directs your attention tothe last sentence as the most important one. You’ll waste less time, and get lessdistracted, if you let the question stems help you out.

• Stick closely to the topic and scope of the stimulus when seeking an inference. Thefurther a choice departs from that which the author takes up, the more likely it is thatthat choice is dead wrong.

12. (D)The stem alerts you in advance that the reasoning is flawed, but given this argument,hopefully you’d recognize the presence of weakness—and the nature of that weakness—even without a hint. Why should the drop in household savings become more pronouncedwhen the population gets older, just because “older people have fewer reasons to save”? Ifthose reasons, although fewer, are stronger than young people’s reasons, then by theauthor’s own logic we should expect to see older people saving more money than kids do.And that’s what (D) points out. The number of reasons people have to save money has littleor nothing to do with the nature of those reasons, and hence nothing to do with who is morelikely to save.

Page 55: LSAT - dl.keywin.orgdl.keywin.org/a/a/aafd9027c6dbefbc457fbda910414daf.pdf · KAPLAN LSAT PREP LSAT RELEASED TEST I EXPLAINED A Guide to the June, 1991 LSAT KAPLAN The answer to the

LSAT PREP ________________________________________________________________ LSAT Test I Explained: Section III

© K A P L A N 53

(A), while true—the author does fail to do those things— fails to take up the older vs.younger contrast that’s at the heart of the argument. What’s truly missing is what (D) isgetting at: the relative strength and weakness of each group’s reasons for saving.

(B) The argument does nothing of the kind. This “negative savings rate” concept is totallyirrelevant to the author’s prediction about whether older people will continue to save lessand less money.

(C) The argument need not cite evidence as to increasing age, because the author’sprediction is hypothetical: “If the average age...continues to rise....” That hypotheticalprediction doesn’t hinge on proving that the average will in fact increase.

(E) brings up an irrelevant distinction, between pre- and post-tax income. That has nothingto do with the strength or weakness of the prediction in question.

• When you’re flat-out told that an argument is faulty, endeavor to articulate toyourself just what that flaw is, prior to looking at the choices. Those extra seconds ofevaluation can really pay off in more right answers and less wasted time.

13. (C)You can tell that the point of the argument comes in the first clause—The pit bull is not abreed—because everything that follows it is signaled by “as do...” and “It is like...,” both ofwhich are reliable signals of evidence, examples, and analogies. Both police dogs andSeeing-Eye dogs—terms that are “like” the term “pit bull,” remember—are of the Germanshepherd breed, we’re told; what differentiates them is what they do. Inferably, then, a “pitbull” could be a dog of any number of breeds, because it’s a designation of what the dogdoes. (C) picks up on this analogy: If “Seeing-eye dog” and “police dog” designate dogsaccording to what they do, and “pit bull” is like those terms in this respect, then a dog canbe designated a “pit bull” based only on what the dog does, irrespective of its breed.

(A) is quite possible, logically speaking, since “pit bull” is a term for a function not abreed. But it’s hardly the point the author is leading toward. The role of the Germanshepherd here is to illustrate the difference between breed and function.

(B) Au contraire, a pit bull is distinguished by what it does and not by appearance alone.

(D) repeats evidence. The phrase “It is like” introduces an analogy, and the last sentence(which is (D), after all) is there to explain what that analogy is all about.

(E) Distinguishing between breeds is totally outside the author’s scope.

• Questions like this one are made much easier if you’re in the habit of readingarguments for their structure. Whenever you read a sentence, ask yourself, “Is it theconclusion? Is it all or part of the evidence? Is it background info, or filler? Or arestatement of something else in the argument?” And speaking of structure . . .

Page 56: LSAT - dl.keywin.orgdl.keywin.org/a/a/aafd9027c6dbefbc457fbda910414daf.pdf · KAPLAN LSAT PREP LSAT RELEASED TEST I EXPLAINED A Guide to the June, 1991 LSAT KAPLAN The answer to the

LSAT PREP ________________________________________________________________ LSAT Test I Explained: Section III

54 © K A P L A N

• Use Keywords. Recognize and interpret the signals that authors use to organize theirprose. Since so many questions hinge on structure, Keywords are reliable signpostsfor saving time and racking up quick and easy points.

14. (E)If, as sentence one says, a marketplace is a necessary condition of a monetary system, and if,as sentence two says, fourth century Mesopotamia lacked marketplaces, then clearlyMesopotamia during that period lacked a monetary system. And that’s (E), plain andsimple. Frankly, a student reading the first two sentences and putting them together mighthave come up with the proper inference on her own, sought it among the choices, andfixed on (E) in record time. . . and if that describes the method that you employed here,well, good work!

(A) The argument leaves open the possibility that other fourth century B.C. populationsbesides Greek cities had monetary systems. Not Mesopotamia, though.

(B) is an improper inference of causation. In fact, since a marketplace is a necessarycondition for a monetary system, it’s impossible to have a monetary system before thedevelopment of a marketplace.

(C) We’re told that both Greece and Mesopotamia traded. With each other? Who knows?

(D) Mesopotamia’s situation post-fourth-century-B.C. is outside the scope. We can’t inferanything about what developed later on.

• Most of the time, the right answer to an inference question will hinge on translatingone sentence of the stimulus, or (as here) combining two sentences. Rarely if ever doyou have to combine all of the data to arrive at your answer. Assume that you can getaway with much less work than that! (See Q. 16, below.)

15. (B)Businesses should introduce some variety into their software systems (that’s theconclusion, signaled by “therefore”), because using only one system leaves a companyintensely vulnerable to vandalism. That logic, however, relies on the assumption thatletting a virus loose in a system is worse than rebuilding a system post virus. (B) asserts thatthat assumption is true, and hence it supports the reasoning. Use the Denial Test to checkit: If (B) is false—if it’s no more expensive to rebuild a system than to prevent a problem—then the wisdom of the author’s advice is seriously in doubt.

(A) is an irrelevant generalization about standardization and its effect on business.

(C) Since the argument is solely concerned with businesses that have compatible systems,(C) is beyond the scope.

(D) The argument is about the damage caused by vandals, not about the inherentdestructiveness of programs. And (D)’s other topic, non-linked computers, is equallyoutside the scope.

Page 57: LSAT - dl.keywin.orgdl.keywin.org/a/a/aafd9027c6dbefbc457fbda910414daf.pdf · KAPLAN LSAT PREP LSAT RELEASED TEST I EXPLAINED A Guide to the June, 1991 LSAT KAPLAN The answer to the

LSAT PREP ________________________________________________________________ LSAT Test I Explained: Section III

© K A P L A N 55

(E) is true. But businesses that do need to share data are the topic here, because they’re theones at risk. (E) neither supports nor weakens the logic.

• The conclusion is what every argument is based on. So if a Keyword signal like“therefore” jumps out at you, pay special attention to what follows. In some cases,you can even jump right to the “therefore” in order to read and paraphrase theconclusion first, and then go on to see how the author reached it.

16. (C)The right answer, (C), is a paraphrase of the stimulus’ fourth sentence: The variations thatthe author is touting will “virtually” eliminate problems, but not necessarily solve ‘em all.

(A) demonstrates a classic fallacy: the fallacy of denying the antecedent. This argument issaying that if variety is introduced, data can be protected. We cannot deduce from that thatshould variety not be introduced, data will be lost. Not necessarily!

(B) According to sentence three of the stimulus, a virus-wielding vandal can destroy“much” of a company’s data. (B) goes too far.

(D) is outside the scope. The topic is businesses that have multiple-linked systems.

(E) comes off sounding more like an au contraire choice rather than a valid inference.Contrary to (E)’s spirit, the author is touting minor software variations as a way to make ittougher to access data—at least for vandals, anyway.

• Keep practicing with, and mastering, the logic of if/then statements. There’s nosingle topic that has as much impact on your LSAT score.

17. (D)Superior conductors (“super-conductors”?) can inspire even top orchestras to workharder, according to the author, but such authority cannot be claimed, it must be earned.How does the conductor earn such authority? Simple—by virtue of his or her artisticinterpretation. If the orchestra respects that interpretation, evidently the musicians willgrant the conductor the authority to intensify the rehearsals in order to realize this vision.But wait a minute . . . What if the artistic interpretation the conductor is “pursuing”remains ambiguous to the musicians of the orchestra until it’s fully realized? Doesn’t thisshort circuit the author’s logic? It sure does: How can the conductor win authority basedon an interpretation that hasn’t yet been realized? In order for this argument to work, theauthor must presuppose that top orchestras can appreciate an interpretation beforebringing it to realization.

(A) For all we know, a superior conductor can use the same interpretation for acomposition with many different orchestras and still inspire each orchestra to work harder.The argument is not damaged by denying (A), so (A) need not be assumed.

Page 58: LSAT - dl.keywin.orgdl.keywin.org/a/a/aafd9027c6dbefbc457fbda910414daf.pdf · KAPLAN LSAT PREP LSAT RELEASED TEST I EXPLAINED A Guide to the June, 1991 LSAT KAPLAN The answer to the

LSAT PREP ________________________________________________________________ LSAT Test I Explained: Section III

56 © K A P L A N

(B) “Never satisfied” is a little strong, don’t you think? There’s no reason why a superiorconductor couldn’t be well satisfied by a performance by a top orchestra, which means that(B) is not necessary to this argument.

(C) goes against the grain of the argument: At least in some cases it takes hard-wonauthority on the part of a conductor to inspire an orchestra to put in additional work.

(E) is out of the scope. The argument concerns the characteristics of the superiorconductor, which is a different issue from whether top orchestras are always led by thoseconductors. For all we know, some top orchestras are led by second-rate conductors.(Would they be “semi-conductors”?)

• Always pay attention to the scope of the argument, and recognize when a choicesubtly brings in some element outside of this scope. The author doesn’t make anyclaims about whether top orchestras in practice (for instance) are always led by first-rate conductors, so (E) strays from the argument and thus need not be assumed.

• Don’t be fooled when the testmakers turn the tables on you in the choices; that is,they switch around some element of the argument and present this new concept in achoice. Here, the passage involves the orchestra’s assessment of the conductor’sperformance. (B), on the other hand, centers on the conductor’s assessment of theorchestra. This inverts the situation in the stimulus, and there’s no way such aninversion need be assumed (that is, presupposed) by the argument.

18. (E)The stem tells us there’s a discrepancy at work in this one, so immediately we should be onthe alert for something that doesn’t seem to add up. Hopefully, you had no problemspotting the supposed inconsistency detailed in the short stimulus. Pared down, here’s thestrange situation: The amount of oil in U.S. proven oil reserves is the same as ten years ago,but oil consumption has increased. One possible explanation is ruled out—no new oilfields have been discovered. So where’s the extra oil coming from? (E), if true, solves themystery: It’s coming from the same proven oil reserves. Thanks to technological advances,these oil fields can now turn out more oil, because some oil previously thought to beunextractable is now considered extractable. “Proven oil reserves” are explicitly defined asthe amount of oil “considered extractable from known fields.” “Considered” here is the key:If this consideration has changed in the way noted in (E), then it’s no longer surprising thatthe proven oil reserves are at the same level as ten years ago even though consumptioncontinues to increase. (E) thus reconciles the apparent discrepancy.

(A) presents an irrelevant comparison. Since the passage—and the supposed discrepancycontained therein—is based exclusively on the level and consumption of domestic oil, anycomparison of domestic oil to imported oil has no bearing on the argument.

Page 59: LSAT - dl.keywin.orgdl.keywin.org/a/a/aafd9027c6dbefbc457fbda910414daf.pdf · KAPLAN LSAT PREP LSAT RELEASED TEST I EXPLAINED A Guide to the June, 1991 LSAT KAPLAN The answer to the

LSAT PREP ________________________________________________________________ LSAT Test I Explained: Section III

© K A P L A N 57

(B) So what if the rate of growth is declining? The fact that consumption is still growing(albeit at a slower rate) preserves the mystery. If consumption is growing at all (no matterhow slowly it’s growing compared to ten years ago), and if there are no new wells, how is itthat the oil level is the same as ten years ago? (B) doesn’t help us resolve the discrepancy.

(C) The only possible thing (C) helps explain is why there have been no new oil fieldsdiscovered recently. As for the seeming discrepancy at hand, no help.

(D) Price? Like (C), (D) offers a possible explanation for a single element of the argument,in this case, the increased annual consumption of oil. The issue here is the amount, or level,of oil, and a decrease in price by itself, while possibly a factor in the increasedconsumption (maybe the decreased price has driven the increased consumption), still hasno effect on the surprising amount of oil in the U.S. reserves.

• To resolve a seeming discrepancy or apparent paradox, the correct choice must hit atthe central element of the unusual or surprising finding or result. Here, the conceptof “amount” is at the heart of the mystery: Given the circumstances, how could therebe the same level of oil now as there was ten years ago? (E) hits at that issue bystrongly suggesting that a greater amount of oil from the proven wells now fits thedefinition in the first sentence. Conversely, choices focusing on other kinds of oil(A) and price (D) can’t help us, and should have been easy choices to axe.

• As mentioned above, (C) and (D) offer possible explanations for certain facts in thestimulus. But that’s not enough; we’re looking for a choice that resolves thediscrepancy. Don’t get sucked into a choice just because it’s relevant to something—the correct choice has to be relevant to the task stated in the stem, which in this caseis to reconcile the apparent discrepancy.

• Definitions are key. If the testmakers go out of their way to define a phrase or term,make sure you keep that definition in mind. Often, noticing such a definition andtaking account of it accordingly will be the key to getting the point. Sure, this seemslike a strange situation—U.S. oil is at the same level as it was ten years ago whiledemand and consumption have increased—but when we focus on the fact that oillevel in this case is strictly defined as the amount considered extractable, it becomeseasier to see how (E) helps explain away the seeming discrepancy.

Page 60: LSAT - dl.keywin.orgdl.keywin.org/a/a/aafd9027c6dbefbc457fbda910414daf.pdf · KAPLAN LSAT PREP LSAT RELEASED TEST I EXPLAINED A Guide to the June, 1991 LSAT KAPLAN The answer to the

LSAT PREP ________________________________________________________________ LSAT Test I Explained: Section III

58 © K A P L A N

19. (C)Maybe you saw the problem with this logic right away: The evidence deals with trainservice and customer satisfaction, while the conclusion jumps to the concept of railroads as“a successful business.” Are these things the same? No—you probably know the style ofthe testmakers better by now. As you’ve most likely noticed, they’re very particular aboutthe way they use words on the LSAT. Here, the scope shifts from the first sentence to thelast. Even though “train service” and “successful business” seem correlated in some way,they’re not the same thing. The author should have explicitly stated that there’s aconnection between the two; without such a stated connection, the argument falls flat. Itseems like an obvious connection, but it’s nonetheless necessary for the argument to work.By relating service and customer satisfaction to the success of the railroad business, choice(C) bridges this narrow gap in the argument. (C) is thus the assumption on which theargument relies.

(A) For all we know, these two types of train service may be closely related. (A) seems tolend support to the assertion in the first sentence, but why the railroads can’t run bothservices at once successfully isn’t discussed and isn’t relevant to the argument as a whole,so (A) need not be assumed.

(B) Prioritization isn’t an issue. All the author does is posit a condition that’s necessary if arailroad is to be a successful business. Thus, the author need not assume that being asuccessful business is the railroad’s first priority; this argument could work even if being asuccessful business was far down on the list of a railroad’s priorities.

(D), far from a necessary assumption, is an unwarranted inference based on the finalsentence. The last sentence expresses the concept of necessity (it’s necessary to concentrateon one service to be successful), while (D) interprets this as a statement of sufficiency (all therailroad needs to do is concentrate on one service to be successful).

(E) goes astray by trying to mix the two service lines, but commuters’ demand for freightservice is an irrelevant issue that in no way plugs the hole in this argument.

• Scope shifts cover a lot of ground. Here’s an example where the author shifts thescope of the argument by introducing a new word or phrase, in this case, “successfulbusiness.” If you noticed the shift from “train service” in the evidence to “successfulbusiness” in the conclusion, you probably had no problem picking out theassumption in (C) that bridges the gap between these two distinct elements of theargument.

• Don’t do the author’s work for her! You may have missed the distinction above ifyou inferred the connection in choice (C) on your own. Sure, it’s common sense thatif you don’t serve customers well, you probably won’t have a successful business.Regardless, making this judgment still entails taking a logical leap, because thisauthor never makes this connection that holds the argument together. It’s theauthor’s job to present as tight an argument as possible, and that includes tying upthe loose ends. If (C) weren’t true (the Denial Test)—and it’s possible theoreticallyfor railroads to succeed without serving customers particularly well—then thisargument would fall apart.

Page 61: LSAT - dl.keywin.orgdl.keywin.org/a/a/aafd9027c6dbefbc457fbda910414daf.pdf · KAPLAN LSAT PREP LSAT RELEASED TEST I EXPLAINED A Guide to the June, 1991 LSAT KAPLAN The answer to the

LSAT PREP ________________________________________________________________ LSAT Test I Explained: Section III

© K A P L A N 59

20. (E)We need to find an inferable hypothesis based on the two-sentence stimulus. We get twoideas in the first sentence: Most U.S. people don’t regard big or small business asparticularly efficient or dynamic, although they do seem to think both forms of businessoffer fairly priced goods and services. However (and this Keyword should always jump offthe page at you), we’re told small business is generally perceived as more sociallyresponsible (“a force for good in society”), while big business, according to most people, isonly socially responsible in times of prosperity. Some inferences are difficult to pre-phrase; you may have found that evaluating the choices was the way to go here. Most of thewrong choices touch on issues that fall outside of this very narrow scope. The correctinference is found way down in (E), and comes from combining the last part of sentence 1with the last part of sentence 2. There are times (non-prosperous periods) when mostpeople believe that big business is not socially responsible even though most peopleregard big business as providing consumers with fairly priced goods and services. Whynot? As (E) puts it, there must be something more to the “social responsibility” of bigbusiness than simply providing consumers with fairly priced goods and services.

(A) goes against the spirit of the passage, which suggests that most people in the U.S. dohave definite views on, and thus give thought to, the value of business to society.

(B) and (C) We have no idea how the negatively perceived “efficiency/dynamism” factorsfrom the first sentence relate to most people’s attitudes regarding the other elementsdiscussed (price, social responsibility), so we can’t infer what would result from changes inbusiness’ efficiency or dynamism or changes in the public’s perception of same.

(D) We know that big business is seen as socially responsible in times of prosperity, but wedon’t know why. This perception may be dependent on the perception that big businessprovides consumers with fairly priced goods and services, but this perception might becaused by something else. So we can’t tell what would happen if big business didn’tprovide consumers with value for their money, and we can’t infer (D).

• Formal logic sneaks around the LSAT in subtle ways. Notice that the correctinference required a translation of the last sentence, based on an understanding ofthe formal logic term “only”: “Perceived as responsible ONLY in times of prosperity”means “NOT perceived as responsible in times of NON-prosperity” (economicdownturns, recessions, depressions, etc.). Make sure you’re adept at interpreting theformal logic terms (only, some, always, most, all, none, and so on) that appearthroughout both the Logical Reasoning and the Logic Games sections.

• The correct answer to an Inference question is a statement that is certainly true (or isvirtually so) based on the stimulus. However, the correct answer need not combineall or even most of the statements in the stimulus. For this reason, it can be difficultto form a pre-phrase to Inference questions. So don’t worry if you don’t have astrong idea of what the correct answer should look like before you hit the choices.Just keep a strong grip on the scope of the argument, and look to eliminate choicesthat depart from that scope. (See the next bullet point.)

Page 62: LSAT - dl.keywin.orgdl.keywin.org/a/a/aafd9027c6dbefbc457fbda910414daf.pdf · KAPLAN LSAT PREP LSAT RELEASED TEST I EXPLAINED A Guide to the June, 1991 LSAT KAPLAN The answer to the

LSAT PREP ________________________________________________________________ LSAT Test I Explained: Section III

60 © K A P L A N

• Eliminating choices in Inference questions is easier if you can recognize what youdon’t know from a given situation. Here, we’re given a host of conditions andperceptions. What happens when these change? “If big business were MOREefficient (B)”; or “if small business were MORE dynamic (C)”, we don’t know.Choices like (B) and (C) should be crossed off as soon as you realize that theydiscuss issues that go beyond those discussed in the passage.

21. (C)We’re looking for an assumption in an explanation, so your first task is to locate theexplanation and the observation it is supposed to explain. The Keyword phrase “This is thereason that . . .” gives the structure away: The first sentence is meant to explain theobservation discussed in the second sentence. Squirrels can run up steep inclines veryquickly whereas large animals slow down while moving uphill. What is it about squirrelsthat gives them the edge? The explanation claims that the energy required to run uphill isproportional to body weight, but the energy available to run uphill is proportional tosurface area. So the animals that have an easier time running uphill have relatively moresurface area as compared to their body weight. How does this apply to the observationabout squirrels? We know that squirrels weigh less than larger animals, and so they don’tneed as much energy to run up hills, but we can also infer that squirrels have less surfacearea than larger animals, and so they have less energy available to do the job. So whilesquirrels have an advantage in one aspect (body weight), they have a disadvantage inanother (surface area). So what else has to be true about squirrels? As (C) puts it, squirrelsmust have a high surface area to weight ratio, which means that they have more surfacearea relative to their body weight. (C) fills in the gap by explaining why the squirrels’disadvantage in surface area is more than compensated by their advantage in body weight.Squirrels may have a little less energy to do the job, but this is more than made up by thefact that larger animals are much, much heavier.

(A) Au-contraire: In the event that large animals weigh more than small animals (which ismost likely the case), the first line of the stimulus supports the opposite of (A).

(B) The issue here is moving uphill. The explanation therefore need not rely on anycomparison of the general speed of small and large animals. Moreover, (B) ignores the surfacearea issue and thus fails to tie together the theory and the phenomenon it’s meant to explain.

(D) and (E) both focus on ratios that are never mentioned in the stimulus (energy output isproportional to surface area; energy needed is proportional to weight), so there’s no waythat either of these can be the assumption on which the explanation depends. Moreover, (E)has the same problem as (A)— “run” is simply too general because the stimulus focuses onmoving uphill.

• Keep the terms of the argument straight: Even though the argument focuses on theconcept of “energy,” it’s crucial that you recognize that two different kinds of energyare discussed, and that you keep the distinction between them in mind. (You may havebenefited by jotting down the words “needed to go up—weight” and “available to goup—area” in the margin to represent the two kinds of energy and their respectivesources. It’s great if you were able to keep this distinction clear in your mind, but don’tunderestimate the power of shorthand to help even here on the Logical Reasoning section.)

Page 63: LSAT - dl.keywin.orgdl.keywin.org/a/a/aafd9027c6dbefbc457fbda910414daf.pdf · KAPLAN LSAT PREP LSAT RELEASED TEST I EXPLAINED A Guide to the June, 1991 LSAT KAPLAN The answer to the

LSAT PREP ________________________________________________________________ LSAT Test I Explained: Section III

© K A P L A N 61

• Visualize in order to make the passage your own. Why not picture a squirrel runningup a tree while an elephant lumbers up a nearby hill? Maybe such a mental picturewould help to solidify in your mind the concept of the ratio between surface area andweight and the consequences of such a ratio on an animal’s efforts to move uphill.

• This question is not a bad question to skip, especially if you were running low ontime and noticed the two-question stimulus, Qs. 24-25, at the end of the section. Twoquestions for the price of one in 24-25 may be preferable to suffering through thesomewhat complex ratios and comparisons here.

22. (E)In trying to absorb the 1980’s brand of selfishness into the broader context of human history,the author employs two radically different uses of the word “selfish,” as (E) correctlypoints out. “Selfish individualism” is certainly different from “selfish concern for the humanspecies”, but the author lumps them together. To show that the 80’s have gotten a bum rap,the author would have to show that the individual kind of selfishness that characterized the80’s has been evident throughout history. Evidence regarding the different form ofselfishness described in the last sentence is no help at all, which is why the argument fallsflat.

(A) The claim that selfishness has been present throughout history certainly is relevant tothe argument; it’s simply not backed up correctly. As we’ve seen, it’s the form of historicalselfishness the author discusses (selfish concern for the human species) that gets him intotrouble. As discussed above, the argument would work nicely if the author offeredhistorical evidence regarding the individual kind of selfishness characteristic of the 1980’s.

(B) No such evidence is necessary—the argument is not predicated on the belief nor is itintended to show that humans act selfishly more often than not. In fact, it boggles the mindto think of a way in which this even could be statistically determined.

(C) Au-contraire—the author states that selfishness has been present throughout history.

(D) The classic outside the scope choice: What do other species have to do with thisargument? Nothing—this is about humans and humans only, so the author need notconsider the behavior of other species.

• Equivocation is the fancy name for a situation in which different meanings of thesame word are used within an argument. Usually, the resulting argument is a flop.You need not know the formal name for this device, but you must recognize thisspecial form of scope shift when it appears on the test.

• In Logical Flaw questions, many of the wrong choices focus on omissions—thingsthe author hasn’t done. Don’t choose one of these choices simply because it may betrue. The test is not “did the author leave this out?” but rather “did the author leavethis out and should he or she have put it in?” Don’t fault an author for leaving outsomething he or she is not obligated to include as part of the argument. Here, (B) and(D) fit this description perfectly, and hopefully you were able to axe these choicesquickly.

Page 64: LSAT - dl.keywin.orgdl.keywin.org/a/a/aafd9027c6dbefbc457fbda910414daf.pdf · KAPLAN LSAT PREP LSAT RELEASED TEST I EXPLAINED A Guide to the June, 1991 LSAT KAPLAN The answer to the

LSAT PREP ________________________________________________________________ LSAT Test I Explained: Section III

62 © K A P L A N

23. (C)We’re given the results of a survey that are used to support a journal’s decision, and we’reasked to find the choice that provides the best evidence that the decision was the correctone. Many strengthening questions involve shoring up a central assumption or recognizingand discounting a potential flaw in the logic. And maybe you saw the problem right off thebat: “Sixty-two percent of those that returned the questionnaire supported that change.” “Ofthose that returned the questionnaire” should have sent up a red flag: Maybe the peoplewho bothered to return the questionnaire are more likely to agree with the changes; maybethose who weren’t thrilled with the proposed change were less likely to reply.Furthermore, maybe the sample size was very small, and thus not a good indicator of thepreferences of the larger potential readership. Any of these problems may very well skewthe results, and would make us less likely to believe that the change will lead to an increasein readership. However, if (C) were true, and the opinions of the entire potentialreadership matched almost exactly the opinions of those that returned the questionnaires,then close to sixty-two percent of the potential market favors the change. In that case,changing to the new format would be more likely to increase readership, and thus (C)would provide evidence that the journal’s decision will have the desired effect.

(A) still doesn’t discount the possibility of a non-representative sample: It could be truethat the questionnaire was sent only to those who for some reason or another are likely tosupport the change. It also leaves open the possibility of a very small sample size, one thatmay not reflect the attitudes of the journal’s larger potential readership.

(B) So? How do we know these changes were successful?

(D) Cost is not the issue—we’re interested solely in whether the change will have thedesired effect, which is increased readership. (Even if this cost cut were passed on to thereader, which theoretically could increase readership, the journal’s decision is based on thegeneral positive reaction to format changes, and thus the argument needs to be strengthenedor weakened on these grounds, not on grounds irrelevant to the survey such as a pricechange.)

(E) could only weaken the argument. If the readers who disliked the old format were morelikely to respond to the questionnaire than those that liked the old format, then the surveyresults would seem to be biased against the old format, which suggests that changing itmay not have been such a good idea after all.

• The sample of people questioned is always relevant when evaluating argumentsbased on surveys or polls. One of the most common logical flaws that appears inarguments containing surveys or polls is the non-representative sample—caseswhere the results cannot necessarily be trusted because the group of people whogenerated the results aren’t representative of the population in general, or at least thepopulation on which the surveyors suggest the survey or poll was based.

• Recognizing an argument’s central assumption will sometimes lead you directly tothe answer, or, other times, may get you there indirectly. Here, answer choice (C)shores up this potential problem and thus supports the journal’s decision byensuring that the same positive feedback would be characteristic of the samplegroup relevant to the journal’s objective.

Page 65: LSAT - dl.keywin.orgdl.keywin.org/a/a/aafd9027c6dbefbc457fbda910414daf.pdf · KAPLAN LSAT PREP LSAT RELEASED TEST I EXPLAINED A Guide to the June, 1991 LSAT KAPLAN The answer to the

LSAT PREP ________________________________________________________________ LSAT Test I Explained: Section III

© K A P L A N 63

• Don’t let wordy question stems throw you; translate them into something that’s moreuser-friendly. Here, we’re essentially asked to strengthen an argument, only in thiscase the argument is phrased in terms of a journal’s decision based on aquestionnaire survey. The survey functions as the evidence, and the decision is likethe journal’s conclusion.

24. (E)We’re looking for a principle with which to defend Shanna’s position against Jorge’srebuttal, so naturally, the first step is to understand their respective positions. The topic isart: Shanna believes that the owner of a piece of art has the ethical right to destroy thatwork if he or she doesn’t like it or if it becomes inconvenient to care for. Jorge narrows thescope to unique works of art, which he believes carry an additional burden—an ownershould not destroy a unique work of art with aesthetic or historical value because such awork belongs to posterity and therefore must be preserved. It’s possible to pre-phrase thekind of principle that will favor Shanna’s position over Jorge’s. It must somehow reflect thenotion that the rights of the individual owner supersede the requirements of posterity, orat least something along those lines. (E), which grants such power to the individual, is aprinciple that reflects this notion perfectly. The general nature of the principle— “do whatthey wish with what is theirs”—covers Shanna’s position entirely; certainly even unique artworks with aesthetic or historical value (the kind Jorge wishes to protect) fall into thecategory described in this choice. (E) thus supports Shanna’s position against Jorge’s, andgets the point for 24.

(A) Jorge posits an obligation for all owners of unique and important works of art. Aprinciple that covers only the “serious student of the history of art” is too narrow and weakto help Shanna reply to Jorge’s mandate. (A) also leaves out the “inconvenience” issue. Theserious students in this principle may still want to destroy a work of art because it hasbecome inconvenient to care for, but Jorge’s position would still effectively forbid this,regardless of whether the principle in (A) were accepted.

(B) Au-contraire: This supports Jorge’s position over Shanna’s.

(C)’s not be as strong as (B), but it too works against Shanna’s position, offering anotherreason why it may not be a good idea to allow art owners free reign to destroy their art.

(D) is mostly irrelevant and could only support Jorge’s position. We can’t tell whetherworks of art would qualify as entities over which no one could rightfully claim absoluterights of ownership, but if works of art did so qualify, then there would be limits toownership rights, which could prohibit the kind of destruction that Shanna findspermissible.

• This is an offshoot of a Strengthen/Weaken question. At the same time, the correctprinciple must strengthen Shanna’s argument while weakening Jorge’s. AllStrengthen/Weaken questions include the phrase “if true” or “if accepted” in thestem; this tells you not to argue with the answer choices or question the likelihood ofeach, but rather to accept each choice as true and concentrate instead on what effect ithas on the argument or arguments in the stimulus.

Page 66: LSAT - dl.keywin.orgdl.keywin.org/a/a/aafd9027c6dbefbc457fbda910414daf.pdf · KAPLAN LSAT PREP LSAT RELEASED TEST I EXPLAINED A Guide to the June, 1991 LSAT KAPLAN The answer to the

LSAT PREP ________________________________________________________________ LSAT Test I Explained: Section III

64 © K A P L A N

• Pay attention to the tone of the choices—notice how strongly worded the rightanswer is: The rights of individuals with regard to their property “must not becompromised . . .” There’s nothing wishy washy about that, and indeed (E) leavesJorge no recourse—the obligation he discusses goes out the window if the principlein (E) is accepted.

25. (A)In simpler terms, the question stem asks us to find the statement amongst the choices thatone person in the dialogue would consider true and that the other person would considerfalse. Understanding what the testmakers are after here helps us to conceive of a logical testfor each choice. Take (A), for example. What would Shanna think of the truth of thisstatement? She would think it was true, right? Sure—if you don’t like it, according toShanna, you’re ethically justified, “for that reason alone,” to destroy it. Would Jorge havean opinion regarding the truth of the statement in (A)? Yes—he would not believe theowner of such a portrait can ethically destroy it “for that reason alone.” Based on hisargument, he would impose another restriction: The painting must not be “a unique workof art with aesthetic or moral value.” If it is, then it should not be destroyed. Shanna wouldagree with the truth of (A), while Jorge would not, which means that (A) is the answer weseek.

(B) Public viewing is beyond the scope of both arguments. The issue here is the right todestroy versus the obligation to preserve.

(C) and (E) are out of the scope. The issue here is whether and under what circumstances itis morally permissible to destroy works of art. Whether intentional damage and destructionhappens often or not, (C), is irrelevant to this question. As for (E), what does legalpermissibility have to do with moral permissibility?

(D) Shanna would agree with this statement even if the piece of sculpture were unique. Wecan infer that Jorge would agree with this statement as well; his mandate for preservationapplies only to unique works of art. So (D) turns out to be an au-contraire choice, as bothparties would seem to agree, not disagree, about the truth of this statement.

• Mind those scope shifts! After two full questions dealing with this topic, the word“legally” in choice (E) should stick out like a sore thumb.

• This is an offshoot of a Point-at-Issue question, but unlike a typical Point-at-Issuequestion, we’re not asked for the specific idea that comes between Shanna and Jorge.Rather, we’re asked to use what we know about the discrepancy in their positions torecognize a statement that one would regard as true and the other would regard asfalse. (That is, after all, what “disagreeing about the truth of a statement” means.Always translate a confusing-sounding or complex stem in order to get it into a formthat’s easier to use.)

• In a double-question stimulus, use your work on the first question to help youanswer the second. You should have had a very clear idea about the debate fromyour work on Q. 24, so it shouldn’t have taken long to jump to the choices and settleon correct choice (A).

Page 67: LSAT - dl.keywin.orgdl.keywin.org/a/a/aafd9027c6dbefbc457fbda910414daf.pdf · KAPLAN LSAT PREP LSAT RELEASED TEST I EXPLAINED A Guide to the June, 1991 LSAT KAPLAN The answer to the

© K A P L A N 65

SECTION IV:

LOGICAL REASONING

Page 68: LSAT - dl.keywin.orgdl.keywin.org/a/a/aafd9027c6dbefbc457fbda910414daf.pdf · KAPLAN LSAT PREP LSAT RELEASED TEST I EXPLAINED A Guide to the June, 1991 LSAT KAPLAN The answer to the

LSAT PREP ________________________________________________________________ LSAT Test I Explained: Section IV

66 © K A P L A N

1. (C)The stem immediately alerts us to the two major elements of the stimulus—a problem anda proposal with which to solve it. It also tells us that the proposal falls short; our job is tofind out why. The problem is the storage of nuclear waste. Currently, waste is stored on-site, but this won’t do for the long term. Since no storage solution is yet forthcoming, somepeople propose that we scrap the entire nuclear system altogether—shut down the presentplants and build no new ones. This is the “solution” referred to in the stem. What’s wrongwith this? It totally ignores a huge part of the problem—the nuclear waste that already existsand that is stored “on-site.” The storage of existing waste, after all, is the problem that thesolution is supposed to address. (C) picks up on this flaw inherent in the proposal.

(A) First of all, we can’t infer that shutting the plants will “prevent” such development;maybe it would spur such development on. Secondly, even if we accept the truth of (A), itdoesn’t point out why the proposal falls short of solving “the problem it addresses”—thatis, it ignores the issue of radioactive waste altogether.

(B) falls prey to the same weakness as (A); it doesn’t address why the solution fails inregard to the original problem, the storage of existing nuclear waste.

(D) is way out there. Not only does it ignore, like (A) and (B), the main issue of currentradioactive waste, but it doesn’t even directly address the proposal or any possibleweakness of it.

(E) is consistent with the evidence in the first sentence—while waste is stored on-site fornow, this is not a good long-range solution, or as (E) puts it, the risks of unsafe disposal liein the future. So? All (E) does is reinforce the notion that prompted the proposal—it in noway explains why the proposal will fail to solve the problem it addresses.

• Scope shifts take many forms; sometimes they’re temporal in nature. Watch out forinstances where authors switch liberally, without warrant, between considerations ofthe past, present, and future. Here, the problem is based in the present: What to doabout existing waste currently stored on-site? But the solution shifts the focus to thefuture—we’ll simply create no new waste—and in doing so ignores a big part of theproblem it supposedly is intended to solve.

• Why do we at Kaplan suggest that you read the question stem first in LogicalReasoning? Just look at what we get out of this stem: We’re alerted to the fact that thepassage discusses a problem, and a proposed solution that falls short. This is a lot ofinformation to have before you get to the passage, and hopefully you used thisinformation to guide your initial reading of the stimulus.

Page 69: LSAT - dl.keywin.orgdl.keywin.org/a/a/aafd9027c6dbefbc457fbda910414daf.pdf · KAPLAN LSAT PREP LSAT RELEASED TEST I EXPLAINED A Guide to the June, 1991 LSAT KAPLAN The answer to the

LSAT PREP ________________________________________________________________ LSAT Test I Explained: Section IV

© K A P L A N 67

2. (A)Here we have lots of formal logic statements hidden in what appears to be a casualargument. Working backwards from the last sentence: ALL Hawaiian fruit flies are thoughtto be descendent from the same one or two ancestral females. Now look at the sentenceright before this: The picture-winged drosophilid is a subset of Hawaiian fruit flies. Thatmeans one and only one thing: All Hawaiian picture-winged drosophilids must thereforehave the same characteristic of the entire group of Hawaiian fruit flies—they must all bebelieved to be descendent from the same one or two ancestral females, choice (A).

(B) We have no way to infer that picture-winged drosophilids are unique to Hawaii; justbecause this species is found there doesn’t prevent it from appearing elsewhere.

(C) distorts the argument by attempting to forge a connection between the first and lastsentences. However, no such connection is indicated, because the scope of the first sentenceis “worldwide” while the last sentence strictly concerns Hawaii. Specifically, it could verywell be that NOT all, or even very few of the 1000 - 2000 worldwide species of fruit fliesshare the Hawaiian fruit flies’ characteristic descent from one or two ancestral females.

(D) takes its thrust from the idea in the second sentence, that Hawaii has the most diversepopulation of fruit flies in the world. Even if 500 new fruit fly species were discovered,how would we know whether Hawaii would lose its fruit fly diversity crown? There’s noway to tell other than by pure conjecture, and we’re in no position for that. (D) is notinferable.

(E) Again, like in (C), we get a connection between Hawaii and the world that is in no waysupported by the stimulus. While (E) certainly could be true, there’s simply no way tologically infer it from the limited and focused information in the stimulus.

• Get past the wordy and cumbersome terminology to see the real relationship amongthe groups in the stimulus. Here, this situation boils down to nothing morecomplicated than this: ALL X (picture-winged drosophilids) are Y (Hawaiian fruitflies). ALL Y (Hawaiian fruit flies) are Z (thought to be descendent from the same oneor two ancestral females). Therefore, ALL X (picture-winged drosophilids) are Z(thought to be descendent from the same one or two ancestral females).

• When you’re asked for an inference based on a formal logic stimulus, don’t besurprised if the answer comes from one sentence or a combination of two sentences.Here, the first two sentences are totally unnecessary to derive correct choice (A). Theydo, however, serve as fodder for the wrong choices.

Page 70: LSAT - dl.keywin.orgdl.keywin.org/a/a/aafd9027c6dbefbc457fbda910414daf.pdf · KAPLAN LSAT PREP LSAT RELEASED TEST I EXPLAINED A Guide to the June, 1991 LSAT KAPLAN The answer to the

LSAT PREP ________________________________________________________________ LSAT Test I Explained: Section IV

68 © K A P L A N

3. (C)Does a feather a bird make? Evidently, the author never considers this question—she takesit for granted that feather = bird. It may seem so obvious to us that we don’t even realizethat the author fails to make the connection between the feather impression (the evidence)and the conclusion that this feather impression signifies the earliest bird. What if, duringthis era, other creatures had such feathers as the one found in the limestone slab? Does theconclusion still hold? Of course not—the feather need not signify the earliest bird found todate, or any bird, for that matter, if it could have come from a different kind of creature.The author must assume (C)—that NO other creatures of this period had such feathers—inorder for her conclusion to stand.

(A) The relation of the feather to the pterosaurs is of no import here—the pterosaurs aremerely mentioned to lend historical context to the story, but don’t really play any majorlogical role in the argument. For the argument to work, the author need not assume such aconnection between the feather and the pterosaurs.

(B) Why would the author necessarily assume anything about earlier birds preceding thebird in question when she argues that this bird may be the earliest bird?

(D), like (B), goes against the spirit of the passage; the author clearly believes that thefeather belonged to some sort of bird.

(E) Awkwardness? If anything, the description of the feather as “fully aerodynamic”suggests the author may believe the opposite to be the case.

• The Kaplan Denial Test is simply a matter of common sense. Did you notice how weemployed the Denial Test in the explanation of correct choice (C) without actuallyinvoking its name? You may have employed this reasoning yourself without evenrealizing you were using the Denial Test. If you thought to yourself, “Yeah, if (C)isn’t true, this whole thing is really shaky”, then that’s exactly what you did. If so,congrats; the more you internalize this form of reasoning, the better off you’ll be.

• When looking for an author’s assumption, keep your eye out for something theauthor takes for granted. “Takes something for granted” is another of way saying“assumes.”

• The correct choice in an assumption question is the one that contains a necessarybut unstated piece of information. Of the four wrong choices, some may beirrelevant—things that may or may not be so, but regardless, provide no necessarysupport for the argument. Even if a choice seems to be “true,” that’s not enough for itto be correct; it must also be necessary to the argument to qualify as an assumption.The last kind of wrong choice is the one that violates the spirit of the passage—onethat seems to suggest the opposite of what the author may believe. Those choices,like (B, (D), and even (E) here, should be the easiest to eliminate—nothing that goesagainst the spirit or tone of the argument can be assumed by the argument.

Page 71: LSAT - dl.keywin.orgdl.keywin.org/a/a/aafd9027c6dbefbc457fbda910414daf.pdf · KAPLAN LSAT PREP LSAT RELEASED TEST I EXPLAINED A Guide to the June, 1991 LSAT KAPLAN The answer to the

LSAT PREP ________________________________________________________________ LSAT Test I Explained: Section IV

© K A P L A N 69

4. (C)The test for the choices is simple. Each choice that supports the conclusion must lendcredence to at least one of the two factors responsible for declining home energyconsumption: reduced standards of living and changes in people’s use of their time. We’relooking for the exception, the choice that doesn’t support the conclusion—that means thechoice that doesn’t conform to either of these two factors, and (C) is the one: Having cheapwork done to improve home heating systems neither reduces one’s standard of living norrepresents a significant change in the use of one’s time. Essentially, (C) provides a differentreason for the decline, one that might even call into question the researchers’ conclusionthat the factors they cite are responsible for “almost all of the decline.”

(A) represents a change in the families’ use of time; moreover, a restricted use of one’shouse may very well also qualify as a reduction in one’s standard of living. The result ofsuch action is that less oil is needed to heat the entire house.

(B) mostly supports the “use of time” factor. During the cold months, more people thanbefore frequent public places, and people tend to spend “longer periods in them.”Obviously, this represents a change in the way people spend their time, and during this“outside the house” time, less oil is needed to heat their houses.

(D) is reduced standard of living. They may be colder, but they’re consuming less energy.

(E) hits on both factors. For those who love long hot showers, cutting back may very wellrepresent a reduction in their standard of living. And in a technical sense, although it maynot be by much, such people are slightly altering the way they spend their time.

• Focus on the task you’re assigned. Here, you’re not looking to support the notionthat home energy consumption has declined—you’re looking for examples thatsupport the researchers’ conclusion that the decline is due almost entirely to thefactors they cite. If you didn’t read critically to ascertain the true argument here, youprobably thought that all the choices, including (C), supported the conclusion.

5. (B)Our tricky little senator employs a common political trick: He presents a view thatcontrasts that of the administration, but then proceeds to argue as if the administration’sview doesn’t exist. Even his rhetoric is a carefully constructed attempt to show that only hisparty’s view exists or matters: “Hooray for common sense . . .”; “Everyone knows that . . .”He then concludes that the tax is dead, because no senator would ever vote for somethingthat will bring about such results—the results that his party predicts but that he now triesto put over as the only possible interpretation. Through all this, Strongwood is hoping thatpeople, especially the voting senators, will forget about the other interpretation of theeffects of the tax, namely, the administration’s interpretation. Clearly, in making hisargument, he is assuming that the senators believe the report of his party, not that of theadministration.

(A) Strongwood’s party believes that reducing the tax would increase the deficit. Nowheredoes he or his party suggest what will happen if the tax is increased.

Page 72: LSAT - dl.keywin.orgdl.keywin.org/a/a/aafd9027c6dbefbc457fbda910414daf.pdf · KAPLAN LSAT PREP LSAT RELEASED TEST I EXPLAINED A Guide to the June, 1991 LSAT KAPLAN The answer to the

LSAT PREP ________________________________________________________________ LSAT Test I Explained: Section IV

70 © K A P L A N

(C) The “common sense” comment is NOT expressly directed at his opponents. He mayimply by this comment that the administration lacks common sense on this issue, but this isonly a subtle jab, not nearly enough to qualify as “name-calling.”

(D) The notion of “popular” never arises—he simply believes the senators won’t vote forsomething that, at least in the view of his party, will have the negative effect of increasingthe deficit.

(E) We don’t know why Strongwood feels his party’s study is superior to that of theadministration’s. As far as we know, he assumes nothing about the relative “objectivity” ofeach. He simply announces the superiority of his party’s study, not the reasons for thesuperiority.

• Don’t read more into the stimulus than is there. “Hooray for common sense” is muchdifferent from “my dumb-as-dirt thumb-sucking opponents lack the common senseto see that . . .” The stimulus would have to sound something like the latter(although probably not as harsh so as to fit LSAT standards) if (C) were meant to becorrect.

6. (D)Successful economies have been the ones that have included a high level of technologicaltraining. Moreover, we’re told in the first sentence that this will continue to be the case.Japan is good in this respect, certainly better than Europe, but even Japan has a highnumber of people lacking essential skills. What can we infer from this? Certainly (D)follows logically: Europe needs to get its technical training act together if it wants a moresuccessful economy. This is borne out by the fact that Europe is lacking in one respect(technological proficiency) that “will continue to be” needed for a successful economy.

(A) offers an unwarranted comparison that we have no way of corroborating based solelyon the evidence in the passage. Engineers aren’t even mentioned by name, so we certainlycan’t infer a comparison like this.

(B) Even with its flaws, what evidence exists that there’s a better country to serve as the rolemodel of technological proficiency and training? (B), without support, is not inferable.

(C) We can practically infer that Japan’s economic success is due to a narrow band ofhighly skilled labor; the last sentence suggests that since so many of the workers are skilledin manual labor only, only a relative few are highly skilled in the new technology. But isthis distribution uncommon? Certainly not; this distribution of skilled vs. unskilled labor istypical in most European countries, and we have no basis to infer it’s much differentanywhere else, since Japan is the cream of the crop in this respect (“the model for this sortof training effort”).

(E) contains another comparison that has no basis in the facts of the passage. The scope ofthe passage includes Europe and Japan, period—we know nothing of the relativeeconomic success of “most other countries.”

Page 73: LSAT - dl.keywin.orgdl.keywin.org/a/a/aafd9027c6dbefbc457fbda910414daf.pdf · KAPLAN LSAT PREP LSAT RELEASED TEST I EXPLAINED A Guide to the June, 1991 LSAT KAPLAN The answer to the

LSAT PREP ________________________________________________________________ LSAT Test I Explained: Section IV

© K A P L A N 71

• Sometimes a small, innocuous looking word or phrase turns out to be the key to thewhole question. “. . . and will continue to be . . .” is such a phrase here. Skim pastsuch phrases at your own risk.

• Keep the scope of the passage always in the forefront of your mind. In this case,choices containing words like “engineers” (A) and “other countries” (E) have “scrapheap” written all over them.

7. (D)When did the inhabitants of Mexico start playing this ceremonial game? They wereobserved playing the game with a rubber ball in 1519, and rubber itself was not used inMexico before approximately the year 1000. So the author concludes that the game musthave been invented between approximately 1000 and 1519. Is that right? We know that thegame couldn’t have been played with a rubber ball before that time, but what if the gamehad not always been played with a rubber ball (the Denial of (D))? If the game could havebeen played with something else (and we’ll leave it to your imagination to determine whatthat other object might be), then the game might have been played well before the year1000, and the author’s argument would fall apart, confirming (D) as a necessaryassumption.

To look at it another way, suppose that someone visited America in 1980 and saw theinhabitants playing baseball with aluminum bats. If it were established that aluminum batswere not used before, say, 1960, then would it be fair to say that baseball could not havebeen played before 1960? Of course not, since the same game could have been playedearlier with the materials that were then available.

(A) and (C) are out of the scope. The question concerns when the game was originated, andnot the popularity of the game (C) or game-playing in general (A).

(B) If (B) were false, then the window in time in which the game could have beenoriginated would be narrower, but still within the limits suggested by the author; thus, theargument would remain intact. Since the denial of (B) does not defeat the argument, (B)cannot be necessary to the argument.

(E) is irrelevant. Cortez’s observation is important because it shows that the game could nothave originated after 1519. Otherwise, there is nothing special about Cortez; there’s noreason to require the rubber dating to have been due to him.

• Watch out for scope shifts between the evidence and conclusion! When you spotthem, you’ll be able to form a powerful pre-phrase. In this case, the evidenceconcerned an observation of inhabitants playing a game with a rubber ball, but theconclusion concerned the origin of the game itself. If you asked yourself whetherthe same game could have been played with different materials, you were home free.

Page 74: LSAT - dl.keywin.orgdl.keywin.org/a/a/aafd9027c6dbefbc457fbda910414daf.pdf · KAPLAN LSAT PREP LSAT RELEASED TEST I EXPLAINED A Guide to the June, 1991 LSAT KAPLAN The answer to the

LSAT PREP ________________________________________________________________ LSAT Test I Explained: Section IV

72 © K A P L A N

• There’s a great deal of synergy between the different Logical Reasoning questiontypes. For example, many different Logical Reasoning question types require you toidentify assumptions. Question 7 was posed as an Assumption question, but itcould have easily been posed as a Strengthen/Weaken the argument question, oreven a Flaw question. So improving your ability to spot central assumptions willhave many benefits. You’ll do better on Assumption questions, to be sure, but you’llalso do better on many of the other question types.

8. (C)The question is whether billboards ought to be torn down. Mary Simms, the rep of thebillboard company, not surprisingly, argues against that notion. She claims that if thebillboards are torn down, “our ability to earn a living will be damaged.” The word “our”in this case refers to the outdoor advertising company, a fact that has eluded Jack Jordan, alocal merchant. Jack argues that the billboards are “hurting our ability to earn a living.”Evidently, in his case the term “our” refers to his specific interest group—the town’smerchants. When he states that the “basis for our business is an attractive community,” he’stalking about the business community in general—we can tell by the next sentence: Uglybillboards are a turn-off to potential shoppers. Jordan has misinterpreted the word “our.”Mary’s claim was not directed towards the business community in general, but towardsher advertising company specifically.

• Many LSAT questions have this format: A claim is made, a response is made, but theresponse in ineffective because the responder has made some kind of error, usuallyshifting the scope of the argument. So when you see a question in which a claim ismet by a response, scrutinize the response carefully to see if it is on point.

9. (E)This is a great question to save for last. You have five, count ‘em five, formal logicstatements, and then you have to combine them with a sixth, found in the stimulus. It isdifficult to pre-phrase an answer in this kind of question since the correct answer couldtake any of a number of forms, so there’s nothing to do but combine the statements, andexamine the choices one-by-one. To make things worse, the right answer turns out to be (E).We know that all Capulets are intemperate (E), since all non-Montagues are intemperateand no Capulet is a Montague.

(A), (B), (C) and (D) all ignore the possibility that there are people who are neitherCapulets nor Montagues, a possibility left open by the rules. Recognizing this possibilityeliminates choice (B) directly. We can deduce that all Capulets are intemperate (which iscorrect choice (E)), and they can all be crossed in love, but for all we know there are somepeople who are neither Capulets nor Montagues that share these qualities as well, whichaxes choices (A) and (D). As for choice (C), we know that all intemperate people are non-Montagues, but as far as we know they might be immune to love-crossing as long as theyare also not Capulets.

Page 75: LSAT - dl.keywin.orgdl.keywin.org/a/a/aafd9027c6dbefbc457fbda910414daf.pdf · KAPLAN LSAT PREP LSAT RELEASED TEST I EXPLAINED A Guide to the June, 1991 LSAT KAPLAN The answer to the

LSAT PREP ________________________________________________________________ LSAT Test I Explained: Section IV

© K A P L A N 73

• Remember, every question is worth the same, but some take more time than others. Ifyou can tell that a question will suck up too much of your time, leave it for last. Ifyou don’t recognize this until you are into the question, don’t fight until the bitterend. Skip it or make an educated guess.

• A valid inference must be true, but need not be based on all, or even most of thestatements in the stimulus.

10. (C)“Therefore” signals the conclusion: “Somehow,” concludes this author, heavy rains mustpromote hurricane formation. How come? Because right after it rains in sub-SaharanAfrica, the U.S. gets hit with hurricanes big time. This logical flaw is classic (meaning thatit goes back to ancient logicians and that it appears frequently on the LSAT). Logicians callit the “post hoc fallacy,” and though you need not know that name, you’ve got to becomefamiliar with the error: It’s the fallacy of assuming that because event A preceded event B,therefore event A must have caused event B. Not necessarily!—the events may not becausally linked at all. Chronology, as they say, does not necessarily mean causality. In thesame way that the stimulus cites the rain as a cause of the hurricanes simply because therain came first, so (C) argues that playing competitive sports must causally relate to one’sentrepreneurial skills, simply because so many entrepreneurs played sports first. Note thatthe concept of causality is totally absent from the other four choices, which wander farafield.

(A) assumes that most people who sleep well are healthy—a dubious assumption, but notthe flaw that the stimulus demonstrates.

(B) is focused on a comparison between long and short city blocks. No such comparison canbe found in the stimulus.

(D) argues opposites: Since sun opens the blossoms, dark must close them. Dubious, sure,but the stimulus features no such opposition.

(E)’s conclusion is a prediction; and it reasons from a general statement about events to aparticular claim about liberalization. The stimulus features neither of these tactics.

• The “algebraic” approach to Parallel Logic won’t always pay off, especially when theargument gets complex. Get in the habit of characterizing prose in general terms.When you characterize the conclusion in (E), for instance, as a prediction, it’s easy todiscard (E) promptly, because the stimulus has nothing to do with predictionwhatsoever.

Page 76: LSAT - dl.keywin.orgdl.keywin.org/a/a/aafd9027c6dbefbc457fbda910414daf.pdf · KAPLAN LSAT PREP LSAT RELEASED TEST I EXPLAINED A Guide to the June, 1991 LSAT KAPLAN The answer to the

LSAT PREP ________________________________________________________________ LSAT Test I Explained: Section IV

74 © K A P L A N

11. (B)The author believes that it is more important to criticize democracies that have violatedhuman rights than it is to criticize dictatorships that have committed more violent offenses.Why? Among other claims, the author indicates that the violations committed bydemocracies are worse (“more reprehensible”) than those committed by dictators, since theactions of a democracy reflect the will of the people. So the violations committed bydemocracies can be more reprehensible even if they are less violent. Then, as choice (B) hasit, it must be possible that some human rights violations can be worse than other humanrights violations that are more violent.

(A) This choice should read “All governments commit some inexcusable andreprehensible acts.” (Emphasis added.) The version of this test that came with the LSDASRegistration and Information book may contain an error, substituting “same” for “some,”but in either case, (A) is not a valid inference. For all we know, at least one government issqueaky-clean.

(C) Even though the author claims that criticism would have more of an effect on ademocracy, that’s no basis for claiming that criticism would have no effect on adictatorship.

(D) is out of the scope. The argument never addresses the relative frequency of humanrights violations.

(E) is an irrelevant distinction. The stimulus never mentions those that merely claim torepresent the will of the people.

• Once again (see Q. 9), the correct answer to an inference question need not relate toevery point made in the stimulus, but it must be necessarily true.

• Errors on the LSAT have been known to happen, but they are extremely rare. Onyour test, if you think you see a problem with the question, you’ve probably missedsomething, especially in Logic Games. In any case, don’t spend too much time onone question. When you are really stuck, make an educated guess and move on.

12. (A)This stimulus presents a classic correlation-causation argument. A study found thatsmokers are more likely to snore than nonsmokers, even though snoring is not particularlycommon in either group. From this, the author of the study concludes that smoking alonecan cause snoring. Not so fast. All we know is that two conditions (smoking and snoring)are connected, and that isn’t enough to conclude that the first condition (smoking) causesthe second (snoring). For all we know, the causal relationship could be the other wayaround—snoring could cause smoking. Although that seems somewhat unlikely in thiscase, nothing rules this possibility out. Alternatively, both conditions could be caused bysome third factor (such as stress), as (A) suggests. Thus (A), if true, would explain howsmoking and snoring are correlated without implying a causal relationship between them.So (A) weakens the argument by providing an alternative explanation of the author’sevidence.

Page 77: LSAT - dl.keywin.orgdl.keywin.org/a/a/aafd9027c6dbefbc457fbda910414daf.pdf · KAPLAN LSAT PREP LSAT RELEASED TEST I EXPLAINED A Guide to the June, 1991 LSAT KAPLAN The answer to the

LSAT PREP ________________________________________________________________ LSAT Test I Explained: Section IV

© K A P L A N 75

(B) provides evidence concerning the causes of smoking, but this argument concerns theeffects of smoking, and whether one of them is snoring. By ignoring smoking’s effects ingeneral, and the issue of snoring in particular, (B) misses the point.

(C) and (D) both address the degree of correlation between smoking and snoring, whereasthe author only addresses the question of whether any causal relationship exists. So (C) and(D) might be relevant to the question of the extent of the connection between smoking andsnoring, but they are irrelevant to this argument, which only claims that smoking can leadto snoring, at least some of the time. Furthermore, choice (D) is practically inferable fromthe evidence stating that snoring is not common among either smokers or non-smokers.

(E) has it backwards. Whereas the credited response suggests that a third condition couldcause both smoking and snoring, (E) suggests that both smoking and snoring cause a thirdcondition. This has nothing to do with the author’s conclusion that smoking can causesnoring, so it in no way weakens the argument.

• The concepts of correlation and causation are long-time favorites on the LSAT, andthis question is a prime example of how they are tested. Remember, the fact that twoconditions (like smoking and snoring) tend to happen together does not show thatone causes the other. The causal relationship might be the other way around, or theremight be no causal relationship between them at all.

• Every question is worth one point, but some questions take less time to answer. Besure to get to them, no matter where they appear in the section. Question 12, forinstance, has a short stimulus and short answer choices, and could certainly havebeen answered in less time than it would take to answer a typical Parallel Reasoningquestion.

13. (B)Will Rogers said “all politics is applesauce,” and this author might agree. The voters arenot getting the information they need to make good decisions, since they never learn aboutthe substantive positions that the candidates take on important issues. Instead, all they getis reports on the moves and countermoves of the campaign advisors. Therefore, theadvisors should get out of the way, so that the press can report on the candidates’ positionson substantive issues. What does that assume? For one, it assumes that the candidates havepositions on substantive policy issues, (B). If they have no positions on substantive policyissues, then getting rid of the campaign advisors won’t help; the voters still won’t know thecandidates’ positions on the issues, since there are no positions upon which a report can bebased.

(A) Chess is mentioned merely to illustrate the author’s point; certainly, chess need not bethe most appropriate analogy in this case in order for the argument to stand.

(C) is irrelevant. The argument does not address the issues of how the substantive issues inthe campaign are determined. Instead, it concerns how the candidates’ positions on thoseissues can be discerned, however they came about.

Page 78: LSAT - dl.keywin.orgdl.keywin.org/a/a/aafd9027c6dbefbc457fbda910414daf.pdf · KAPLAN LSAT PREP LSAT RELEASED TEST I EXPLAINED A Guide to the June, 1991 LSAT KAPLAN The answer to the

LSAT PREP ________________________________________________________________ LSAT Test I Explained: Section IV

76 © K A P L A N

(D) would weaken the argument. If the voters don’t care enough about the election to payattention, then it wouldn’t matter whether the campaign advisors stayed out of thelimelight.

(E) conflicts with the stimulus, which indicates that the press is reporting on the politicalprocess and ignoring substantive issues. If the press can attend to one and ignore the other,then they must be different, and so (E) is inconsistent with the evidence.

• Many LSAT stimuli present two alternatives, but discuss only one of them.Whenever that happens, the central assumption in the argument will likely dependupon the characteristics of the other, unmentioned alternative. For example, in thisquestion the author explains that reports on the activities of campaign advisors isnot providing enough information to the voters, and argues that those advisorsshould stay out of the way so that press coverage can shift to the candidatesthemselves. But we don’t know anything about the candidates yet, and so we have toassume something about them if the author’s argument is to hold water.

• Use the Kaplan Denial Test to confirm that a choice is a necessary assumption. If thenegation of a choice will defeat the argument, then that choice must be necessary tothe argument. On the other hand, if a choice can be false without damaging theargument, then that choice cannot be an assumption.

14. (E)Next we’re asked to find a valid inference. In a complicated stimulus like this one, it can bedifficult to pre-phrase an answer to an inference question, so there’s nothing to do butattack the choices. If the press reports only on campaign advisors, and the voters do notcurrently have enough information to make informed decisions, then those reports are notproviding all of the information that the voters need to make those decisions, (E). Ifreporting on the campaign advisors alone was doing the job, then the voters wouldn’t be inthe “information-deprived” situation described in the stimulus.

(A), (C), and (D) are out of the scope. The preferences of the candidates with respect tomedia coverage, (A), their beliefs about the political process, (C), and the relative ease ofdifferent subjects of reporting, (D), are all issues that are never mentioned.

(B) has a few problems. First, it is the campaign advisors who are in the limelight, not thepress. Second, similar to the other wrong choices mentioned above, we’re not givenenough information to infer anything about what the press enjoys.

• Focus on the scope of the argument, and eliminate choices that depart from it. Stickto the argument that you are given, and don’t be distracted by irrelevant issuesbrought up in the answer choices.

• Sometimes an answer choice will make a claim about someone’s intention or beliefs,even though the stimulus describes only their behavior. In this question, choices(A), (B), and (C) all commit this scope shift. These choices can be tempting when theclaim about the intention or belief could be true (or is often true in the real world).

Page 79: LSAT - dl.keywin.orgdl.keywin.org/a/a/aafd9027c6dbefbc457fbda910414daf.pdf · KAPLAN LSAT PREP LSAT RELEASED TEST I EXPLAINED A Guide to the June, 1991 LSAT KAPLAN The answer to the

LSAT PREP ________________________________________________________________ LSAT Test I Explained: Section IV

© K A P L A N 77

Read critically and don’t infer anything about beliefs or attitudes without concretesupport from the text.

• The Kaplan Denial Test can be used to check the choices in Inference questions, too!Remember that a valid inference must be true given the information in the stimulus.So any choice that could be false cannot be a valid inference. Here, if “reporting oncampaign advisors” was providing enough info for the voters to make informeddecisions, the argument takes a nose dive.

15. (E)Most humans are right-handed, but what about animals? Are animals equally likely tohave a preference for their left limbs as their right limbs? Certain studies say that animalswill be equally likely to be “right-handed” as “left-handed,” but the author disagrees. Insupport, the author notes that dogs often “shake hands” with their right paw. OK, so what?Suppose, as (E) suggests, that dogs are influenced by their masters in learning to performtricks like “shaking hands.” In that case, we would expect that since most masters(presumably humans) are right-handed, most dogs would be taught to perform tricks in a“right-pawed” manner. So if (E) is true, it isn’t the dogs that are showing a preference fortheir right limbs at all. Rather, dogs that “shake hands” with their right paws are merelyreflecting the preferences of their masters.

(A) and (D) are out of the scope. We’re asked to explain why the “shaking hands” evidencedoes not weaken the claim that animals will be equally likely to favor their right limbs asopposed to their left limbs. Answer choices (A) and (D) completely ignore the curiousfindings regarding “shaking hands,” and so cannot help in a defense against this supposedcounterexample. Choices (A) and (D) do provide some evidence that dogs show nopreference for their right limbs in other respects, which might help counter the claim thatdogs have a tendency to be “right-handed” in all respects, but that’s the subject of anotherquestion.

(B) The evidence is unrepresentative, but not for the reason in (B). There’s no grounds forbeing especially suspicious of “front-paw-only” behavior.

(C) implies that dogs are exempt from the kinds of societal pressures and inconveniencesfacing left-handed humans. Inferably, then, dogs aren’t pressured in any way to shakehands with their right paw—they simply do so naturally. Thus, if anything, (C) strengthensthe validity of the counterexample by ruling out an alternative explanation for the dogs’noted behavior. (Notice how this differs from correct choice (E), which provides analternative explanation for why dogs usually “shake hands” with their right paw, and thusdamages the validity of the counterexample.)

• Whenever a survey is used as evidence, think “unrepresentative sample.” Look forsomething about the evidence that suggests bias or unreliability.

• Pre-phrase whenever you can. You’ll save time, and you’ll be less likely to fall fordistracting answer choices. Why spend time figuring out why the four wronganswers are wrong when it’s easier and faster to recognize the one that’s right?

Page 80: LSAT - dl.keywin.orgdl.keywin.org/a/a/aafd9027c6dbefbc457fbda910414daf.pdf · KAPLAN LSAT PREP LSAT RELEASED TEST I EXPLAINED A Guide to the June, 1991 LSAT KAPLAN The answer to the

LSAT PREP ________________________________________________________________ LSAT Test I Explained: Section IV

78 © K A P L A N

• Always read the question stem first! You’ll get valuable information about your task,and you may even get a preview of the topic and scope of the argument.

16. (A)Reading the question stem first pays off big here. We’re looking for what the student(mistakenly) thought the professor said. The student claims to have a counterexample to theprofessor’s “genetic law.” She has Type O blood, even though her father does not haveType O blood. What claim would this counter? It would certainly show that it is notnecessary for a Type O child to have a Type O parent. So choice (A), claiming that onlypeople with Type O blood can have children with Type O blood, would be disproved bythe student’s rejoinder. For this reason, (A) must represent the student’s misinterpretationof the professor’s remark.

(B) If (B) were the answer, the student would have to have said something like “My fatherhas Type O blood and I have Type B blood.”

(C) is consistent with the student’s claim, so (C) could not be the claim to which the studentobjects.

(D), (E) What other children? What other parent? The student never mentions either ofthem, so there’s no basis for concluding that the student is thinking of either one.

• Always read the question stem first. Sometimes, you get a sneak preview of theargument’s topic and scope, and you always get information to direct your attack onthe stimulus.

• When the second speaker has misinterpreted the first speaker’s statement, focus onthe second speaker’s position. Think about the claim that the second speaker mustbe countering; since there’s been a misinterpretation, expect that this claim won’tmesh with the first speaker’s actual statement.

17. (B)Here we get another curious situation: R-Bar Beans sell fewer cans nationally than itscompetitors, even though R-Bar Beans are preferred among all age groups in a recentsurvey. This seems odd, because we would normally expect that people would buy moreof a product that they find superior. So what’s going on? Four of the choices, if true, wouldhelp resolve the discrepancy, and one of them (the credited response) would not. If (B)were true, and the preference for R-Bar Beans is more pronounced in some groups asopposed to others, then we would expect R-Bar Beans to have more success in those R-Barfavorite groups. However, we would still expect R-Bar Beans to have the overall lead inbean sales, since R-Bar is preferred among all age groups. That isn’t the case, as R-Bar ranksat best third in sales. (B) adds nothing to our understanding of how R-Bar compares withthe other brands, and thus doesn’t help us solve the mystery.

(A) If the other beans are cheaper than R-Bar Beans, then it’s not surprising that somepeople buy them even if they think R-Bar Beans taste better. It’s no shock that Hondasoutsell Rolls Royces, even though most people would prefer to drive a Rolls.

Page 81: LSAT - dl.keywin.orgdl.keywin.org/a/a/aafd9027c6dbefbc457fbda910414daf.pdf · KAPLAN LSAT PREP LSAT RELEASED TEST I EXPLAINED A Guide to the June, 1991 LSAT KAPLAN The answer to the

LSAT PREP ________________________________________________________________ LSAT Test I Explained: Section IV

© K A P L A N 79

(C) If the survey is not representative of the national market, and if R-Bar Beans are noteven available nationwide, then it’s no wonder that R-Bar isn’t #1.

(D) If many stores refuse to carry R-Bar, then reduced sales for R-Bar would be expected.

(E) solves the mystery by telling us that R-Bar sales figures reflect only three months ofsales as compared with a full year of sales for the two competitors. No wonder R-Bar soldfar fewer cans of beans last year than the others even though the recent survey shows thatcustomers prefer R-Bar. Once R-Bar does catch on, though, there’s a good chance that TexasT and Aunt Sally’s will be “has-beans.”

• The first step in Paradox questions is identifying the apparent paradox. Once youunderstand why the result was unexpected, you should have a strong grip on thescope of the argument, which will help you determine what will resolve the apparentparadox.

• Accept the evidence that you’re given, but be skeptical about the gap between theevidence and the conclusion. Any issue ignored by the author is fair game.

• “Each of the following EXCEPT” questions can be very time-consuming. If you’reshort on time, you may want to tackle other questions first.

18. (E)Some cosmetics firms claim that the new product-safety tests, which use cultures of humancells, reduce the need to conduct tests on live animals. Why would the firms devoteresources to perfecting the tests? They must attach some value to reducing the scope oflive-animal tests, all other things being equal. Therefore, the managers of those firms, theones making the decisions, must believe that as long as safety measures can be maintained,it is better not to perform tests on live animals, choice (E).

(A) has two problems. First off, “pressure” is too strong a word to fit the scope of thisstimulus. All we know is that the firms are committed to the concept of the new product-safety tests that will reduce the need for testing animals. We can’t rightfully infer from thisthat the firms are being pressured into this commitment. But even if we do infer this outsidepressure, there’s still no way to conclude who initiated such pressure.

(B) Consumers and their preferences are one step removed from the information in thestimulus. There’s no way for us to conclude which products consumers are more likely tobuy.

(C) Just like “consumer preferences” in (B) above, financial consultants’ beliefs regardingthe costs associated with the new product-safety tests are outside the scope of theargument.

(D) It is impossible for us to determine the number of product tests necessary using thenew system as opposed to the old. We simply aren’t given enough information to validatethe researchers’ beliefs stated in (D).

Page 82: LSAT - dl.keywin.orgdl.keywin.org/a/a/aafd9027c6dbefbc457fbda910414daf.pdf · KAPLAN LSAT PREP LSAT RELEASED TEST I EXPLAINED A Guide to the June, 1991 LSAT KAPLAN The answer to the

LSAT PREP ________________________________________________________________ LSAT Test I Explained: Section IV

80 © K A P L A N

• Don’t let your outside knowledge interfere with your handling of the questions.Sometimes, a question, stimulus, or passage will touch on an issue about which youhave a personal opinion. Always stick to the facts as they appear on the page.

• Pay strict attention to the scope of the passage, and you should be able todifferentiate between what you know and what you don’t know from any givenstimulus. Here, (A) through (D) discuss issues that are at least one step removedfrom the thrust of the short passage. Therefore, there’s no way any of these choicescan serve as a valid conclusion.

19. (A)When the testmakers give you a question that can be decoded quickly with Keywords andstructural clues, thank them, get the right answer, and move on. We’re asked for the mainpoint of the passage, and so you’ll want to key in whenever the author expresses heropinion. The first sentence merely asks if there is a necessary relationship between negativefeelings toward the elderly and plucking out gray hairs. The second sentence begins withthe Keyword “unless,” signaling an exception, followed by the strong statement of opinion“there is no necessary connection.” The third sentence makes a distinction betweenattitudes that supports the notion that there is no necessary connection. The fourth sentencebegins with the Keyword “furthermore,” signaling additional supporting evidence, andthe fifth sentence begins with the Keyword “but,” signaling a contrast to the supportingevidence in the previous sentence. Neither of these sentences can be the main point. Sowhere are the expressions of opinion? The second sentence is the only qualifier. It claimsthat it would be premature to conclude that there is a necessary connection between hair-plucking and having negative attitudes about the elderly, which is paraphrased by choice(A).

(B) is a statement with which the author would agree, but the differing evaluations of theseattitudes is used to support the author’s argument that there is no necessary connectionbetween them. (B) is a step along the way, but not the final destination.

(C) The two claims mentioned in (C) are not linked to each other by the passage. They areboth used to support a further point, the view that there is no necessary connectionbetween disliking aging and disliking the elderly.

(D) brings in a new term, “fine,” without support from the stimulus. Furthermore, “beingelderly” is not part of the distinction drawn in the stimulus—that distinction is betweenpeople’s attitude toward the elderly and their feelings about the process of getting oldthemselves.

(E), like (B), is a view with which the author would agree, but as discussed above, theauthor’s moral arguments are used to support the main point that there is no necessaryconnection between these two attitudes.

• Use Keywords and structural signals to help you navigate through LogicalReasoning stimuli and Reading Comprehension passages. To find the main point,watch out whenever the author expresses an opinion, and pay less attention tosupporting evidence.

Page 83: LSAT - dl.keywin.orgdl.keywin.org/a/a/aafd9027c6dbefbc457fbda910414daf.pdf · KAPLAN LSAT PREP LSAT RELEASED TEST I EXPLAINED A Guide to the June, 1991 LSAT KAPLAN The answer to the

LSAT PREP ________________________________________________________________ LSAT Test I Explained: Section IV

© K A P L A N 81

• Many Main Point answer choices mention elements that were present in thestimulus, but were only a step along the way to the author’s real conclusion. So don’tjust choose the first choice that contains something you recognize. Ask yourself ifthat information was itself used as evidence to bolster another larger point.

20. (D)Next we’re asked to find the one method of argument out of five the author does not use.While the author does criticize those who hold negative attitudes about the elderly, theauthor does not “discredit a common stereotype about the elderly,” choice (D). Theauthor does imply that aging is associated with impaired eyesight and hearing, but farfrom discrediting this information, the author uses it in showing why disliking thosefeatures may not be associated with disliking the elderly.

(A) In the first sentence, the author points to the lack of research to support the assertionthat plucking gray hairs and harboring negative attitudes toward the elderly areconnected.

(B) In pointing to a lack of evidence, applying general principles, and in making adistinction, the author could fairly be described as appealing to reason. Further, it’s hardto imagine what this stimulus would have to look like for (B) to be the right answer. Only avery sketchy (perhaps even circular) argument doesn’t appeal to reason in some way oranother.

(C) In the fourth sentence, the author cites the general principle that having negativeattitudes toward the old because they are old is immoral.

(E) In the third sentence, disliking the elderly is distinguished from disliking conditionsassociated with the aging process.

• While working on Method of Argument questions, get to know the “usualsuspects,” the choices that appear in many questions. Pointing to the lack ofevidence on the other side (A), appealing to general principles (C), and makingdistinctions (E), are common argumentative techniques on the LSAT.

21. (D)The valid inference we’re asked to find is merely the contrapositive of one of the statementsin the fourth sentence. The author states that “a society that has no laws has no crimes.” In“if-then” format, this means “if a society has no laws, then that society has no crimes.” Toform the contrapositive of any if-then statement, reverse the terms and negate (deny) both.To deny that a society has no laws is to claim that the society has at least some laws, and todeny that a society has no crimes is to claim that the society has at least some crimes. Soreversing the terms and negating both leaves us with “a society that has some crimes hassome laws,” which is choice (D).

(A), (B) No, crimes guarantee laws, not the other way around. For all we know, at least onelawful society is crime-free. (Did you notice that these two choices are contrapositives ofeach other?)

Page 84: LSAT - dl.keywin.orgdl.keywin.org/a/a/aafd9027c6dbefbc457fbda910414daf.pdf · KAPLAN LSAT PREP LSAT RELEASED TEST I EXPLAINED A Guide to the June, 1991 LSAT KAPLAN The answer to the

LSAT PREP ________________________________________________________________ LSAT Test I Explained: Section IV

82 © K A P L A N

(C) and (E) introduce a new term, “many,” without support from the stimulus. For all weknow, a society could have many laws without any crimes at all (C), and it is also possiblethat a crime-plagued society has few laws, (all the criminals could be breaking the samelaw, for instance), which axes (E).

• Know your contrapositives! You’ll need them in Logic Games, and in LogicalReasoning as well. Many correct answers to inference questions are based on thecontrapositive of if-then statements.

• Whenever you are given a statement in if-then form, you may safely infer thecontrapositive of that statement. To form the contrapositive of an if-then statement,reverse the terms and negate both.

22. (B)Is there a connection between the amount of rain in the Galapagos and the size of finchspecies that tend to thrive there? Finches eat seeds, and the amount of rain helps todetermine the kind of seeds that are present. In times of little rain, large finch species havethe upper hand, since only they have bills large enough to crack the large, hard seeds.However, rainy times favor plants with small seeds, which means the small finch specieshave plenty to eat, as their inability to crack the large, hard seeds is no longer so significant.However, the large finches cannot eat fast enough to meet their extraordinary energyrequirements. Why would that be? The author must be assuming (B), that during rainyweather, fewer large, hard seeds are being produced. After all, if the supply of large, hardseeds were as high as ever in rainy times, then the large species should be able to thrive,since that same level of large, hard seeds was enough to allow the large species to thrive intimes of less rain. So if rain really does play a role in which finch species thrive, then theauthor must be assuming (B).

(A) mixes up terms. The author describes large, hard seeds, and small seeds, but there’s notelling what difference small, hard seeds could make.

(C) Individual finches could stay about the same size during rainy periods withoutinjuring the argument. If they did grow to big-bird size during those periods, they wouldencounter the same problems as the larger species, but there’s no reason to believe that theywould encounter such a growth spurt.

(D) is irrelevant. The argument concerns what happens when the climate changes, and notthe relative frequency of different climates.

(E) For all we know, all species have the ability to crack the small seeds, so there’s no needto assume that the small seeds can be digested without cracking.

• Read the question stem first. Sometimes, as in this question, you are told the topicand even the scope of the argument. It pays to know this information before youtackle the stimulus.

Page 85: LSAT - dl.keywin.orgdl.keywin.org/a/a/aafd9027c6dbefbc457fbda910414daf.pdf · KAPLAN LSAT PREP LSAT RELEASED TEST I EXPLAINED A Guide to the June, 1991 LSAT KAPLAN The answer to the

LSAT PREP ________________________________________________________________ LSAT Test I Explained: Section IV

© K A P L A N 83

• Don’t “fill in the blanks” for the author. Anything necessary to the conclusion that isleft out of the argument must be an assumption. Sometimes, an assumption is sobasic that it is easy to overlook, but this kind of assumption is often the key to theright answer.

23. (C)We’re looking for a situation that would favor Fring’s view over Blatt’s view. Blatt thinksthe opinions offered by consultants are worth the money. Fring believes that consultantsare hired so that executives will have someone to blame when things go wrong, and (here’sthe key phrase) executives prefer to pay more for consultants, since increased pay meansincreased responsibility. If (C) were true, and a reduction in consulting fees led to areduction in business, then Fring’s claim that executives prefer to be over-charged wouldbe strengthened.

(A), (B) The issue is the reason for hiring a consultant. Without knowing the cost of aconsultant or the value associated with his or her advice, the decision to hire or not hire aconsultant does not favor either speaker.

(D) would strengthen Blatt’s position, since a link between consultant fees and moneysaved by the company suggests that consultants have some worth after all.

(E) We can’t tell if the money paid to the consultant was worth it (since the stores aremarginally profitable at first), and we don’t know whether the company hired theconsultant to help make decisions or just to have someone to accept any future blame, soit’s hard to see how (E) helps either side.

• Get a strong grip on the scope of the argument before you hit the choices. If youknow the real point at issue, then you’ll zero in on the choice that impacts on thatpoint in the proper way.

Page 86: LSAT - dl.keywin.orgdl.keywin.org/a/a/aafd9027c6dbefbc457fbda910414daf.pdf · KAPLAN LSAT PREP LSAT RELEASED TEST I EXPLAINED A Guide to the June, 1991 LSAT KAPLAN The answer to the

LSAT PREP ________________________________________________________________ LSAT Test I Explained: Section IV

84 © K A P L A N

24. (E)Here’s another Parallel Logic example readily reduced to algebra: All X (contemporaryads) are Y (efforts to persuade). Some X (some contemporary ads) are Z (morallyreprehensible). Therefore some Y (efforts to persuade) are Z (morally reprehensible). Incorrect choice (E), sonnets = X, short poems = Y, and thematically pluralistic poems(whatever that means) = Z.

(A) contains only two pieces of evidence, not the requisite three; and (A)’s “No X are Y”evidence has no parallel in the stimulus.

(B) and (D), like (A), fail to provide the requisite three pieces of evidence. Moreover, (B)’sconclusion is about “reliability,” a concept that is never mentioned in the evidence (noticethat in both the stimulus and correct choice (E), both terms in the conclusion do appear inthe evidence). And (D)’s conclusion is that “All X are Y”—very different from the stimulus.

(C) All X (good managers) are Y (make important decisions), while some Z (managers ingeneral) are not Y (fail to make important decisions). Therefore some Z are not X. Way farafield from the stimulus.

• In Parallel Logic, watch for the correct answer to rearrange (as (E) does here) theorder in which the evidence and conclusion are presented. The placement of thepieces is a stylistic decision, not a logical one. As long as (E) has a “Some X are Z”conclusion—which it does—then its conclusion is parallel to that of the stimulus,irrespective of where in the paragraph the conclusion appears. Ditto for theevidence: The logic is the same even if the premises are presented in a differentorder.

Page 87: LSAT - dl.keywin.orgdl.keywin.org/a/a/aafd9027c6dbefbc457fbda910414daf.pdf · KAPLAN LSAT PREP LSAT RELEASED TEST I EXPLAINED A Guide to the June, 1991 LSAT KAPLAN The answer to the

I.N. LL3122 Rev.A Printed in the USA